Онлайн вычисление производной функции: Экстремумы функции онлайн

Содержание

Экстремумы функции онлайн

С помощью данного сервиса можно найти наибольшее и наименьшее значение функции одной переменной f(x) с оформлением решения в Word. Если же задана функция f(x,y), следовательно, необходимо найти экстремум функции двух переменных. Также можно найти интервалы возрастания и убывания функции.
  • Решение онлайн
  • Видеоинструкция
  • Оформление Word
  • Также решают
Найти наибольшее и наименьшее значение функции

y =

на отрезке [;]

Включать теорию

Правила ввода функций:

Вместе с этим калькулятором также используют следующие:
Асимптоты функции

Уравнение касательной к графику функции

Построение графика функции методом дифференциального исчисления


Экстремум функции двух переменных

Вычисление пределов

Необходимое условие экстремума функции одной переменной

Уравнение f’0(x*) = 0 — это необходимое условие экстремума функции одной переменной, т. е. в точке x* первая производная функции должна обращаться в нуль. Оно выделяет стационарные точки xс, в которых функция не возрастает и не убывает.

Достаточное условие экстремума функции одной переменной

Пусть f0(x) дважды дифференцируемая по x, принадлежащему множеству D. Если в точке x* выполняется условие:

f’0(x*) = 0
0(x*) > 0

то точка x* является точкой локального (глобального) минимума функции.

Если в точке x* выполняется условие:

f’0(x*) = 0
0(x*) < 0

то точка x* — локальный (глобальный) максимум.

Пример №1. Найти наибольшее и наименьшее значения функции: на отрезке [1; 3].
Решение.

Критическая точка одна x1 = 2 (f’(x)=0). Эта точка принадлежит отрезку [1;3]. (Точка x=0 не является критической, так как 0∉[1;3]).
Вычисляем значения функции на концах отрезка и в критической точке.
f(1)=9, f(2)=5/2, f(3)=3 8/81
Ответ: fmin=5/2 при x=2; fmax=9 при x=1

Пример №2. С помощью производных высших порядков найти экстремум функции y=x-2sin(x).
Решение.
Находим производную функции: y’=1-2cos(x)

. Найдем критические точки: 1-cos(x)=2, cos(x)=½, x=±π/3+2πk, k∈Z. Находим y’’=2sin(x), вычисляем , значит x=π/3+2πk, k∈Z – точки минимума функции; , значит x=-π/3+2πk, k∈Z – точки максимума функции.

Пример №3. Исследовать на экстремум фцнкцию в окрестностях точки x=0.
Решение. Здесь необходимо найти экстремумы функции. Если экстремум x=0, то выяснить его тип (минимум или максимум). Если среди найденных точек нет x = 0, то вычислить значение функции f(x=0).
Следует обратить внимание, что когда производная с каждой стороны от данной точки не меняет своего знака, не исчерпываются возможные ситуации даже для дифференцируемых функций: может случиться, что для сколь угодно малой окрестности по одну из сторон от точки x0 или по обе стороны производная меняет знак. В этих точках приходится применять другие методы для исследования функций на экстремум.

Пример №4. Разбить число 49 на два слагаемых, произведение которых будет наибольшим.
Решение. Обозначим x — первое слагаемое. Тогда (49-x) — второе слагаемое.
Произведение будет максимальным: x·(49-x) → max
или
49x — x2

Наибольший объем цилиндра

Найти размеры цилиндра наибольшего объема, изготовленного из заготовки в форме шара радиуса R.
Решение:

Объем цилиндра равен: V = πr2H
где H = 2h,
Подставим эти значения в целевую функцию.

V → max
Найдем экстремум функции. Поскольку функция объема V(h) зависит только от одной переменной, то найдем производную с помощью сервиса
Производная онлайн
и приравняем ее к нулю.
dV/dh = 2πR2 — 6πh2
dV/dh = 0
2πR2 — 6πh2 = 0 или R2 = 3h2
Откуда


При высоте и радиусе основания размеры цилиндра будут наибольшими.

Тесты по теме «Производная» онлайн

  • Производная. Формулы.

    25.10.2020 7671

    Данный тест будет полезным  учителю для осуществления быстрого контроля на уроке, а также ребятам, которые желают проверить свои знания по данной теме.

  • Производная и ее применение

    25.12.2019 7918 0

    Данный тест создан для проверки знаний по темам «Функция», «Производная»

  • Производная сложной функции.

    31.10.2020 5061 0

    Данный тест будет полезным  учителю для осуществления быстрого контроля на уроке, а также ребятам, которые желают проверить свои знания по данной теме.

  • Производная функции (высшая математика)

    07. 12.2021 237 0

    Тест по курсу высшей математики на тему «Производная функции». Тест создан с целью проверки элементарных теоретических знаний производной и умения решать простейшие задачи производной.

  • Нахождение производной суммы. Формулы дифференцирования.

    18.04.2020 1902 0

    Использование формул дифференцирования и правила суммы для нахождения производных.

  • Нахождение производной

    29.02.2020 3380 0

    тренировочный тест на нахождение производной сложной функции, производная частного и произведения

  • Производная элементарных функций

    18. 04.2020 10936 0

    Перед Вами тренировочный тест, проверяющий усвоение небольшой, логически завершенной части темы «Производная». Содержание и уровень сложности включенных в него заданий, в основном, отвечают обязательным требованиям к математической подготовке студентов, обучающихся по специальностям технического профиля. Планируется, что на выполнение этого теста Вы потратите не более 5 минут.

  • Математический анализ. Вычисление производных.

    12.03.2021 172

    0

    Данный тест рассчитан на выпускников школ и первокурсников ВУЗов. Проверьте свои знания в теории производных.

  • Производные элементарных функций

    16. 11.2018 1307 0

    Тест рекомендуется использовать при изучении темы «Производные элементарных функций» по учебнику Ю.М. Колягина, 11 класс

  • Наибольшее и наименьшее значения функций

    23.12.2018 1771

    Контрольный тест по алгебре и началам анализа в 11 классе. По материалам профильного ЕГЭ 2019 года (№12)

  • Применение производной к исследованию функций в заданиях ЕГЭ №2

    15.01.2021 826 0

    Тест  для обучающихся средней школы,  предназначен для подготовки к ЕГЭ и  проверки уровня знаний по  теме «Физический, геометрический смысл призводной и применение производной к исследованию функций «.

  • Тест «Вторая производная. Физический смысл производной»

    05.06.2022 5 0

    Тест «Вторая производная. Физический смысл производной». Тест по теме «Вторая производная. Физический смысл производной» проверяет знание определения первой и  второй производной, понимание физического смысла первой и второй производной функции, умения вычислять первую и вторую производные функций. умения вычислять значения производных функций в точке. Тест содержит 10 вопросов с выбором единичного верного ответа и вопросы с вводом верного ответа в виде числа.

  • Производная. Геометрический смысл производной.

    05. 06.2022 26 0

    Алгебра и начала анализа. Тема «Производная» Тест по теме «Производная. Геометрический смысл производной».

  • Производная функции в школьном курсе математике

    19.09.2018 1317 0

    Тест состоит из 10 вопросов раскрывающих тему «Производная функции»

  • Математика тест для 11 класса по теме производная

    30.01.2019 312 0

    Тест предназначен для учащихся 11 классов или студентов 1 и 2 курса. Содержит задания по теме «Производная и её приложения», «Интеграл», «Первообразная», «Пределы». Включает в себя 19 заданий

  • Тест для обучающихся 11 класс в форме ЕГЭ (профильный уровень)

    08.02.2020 9 0

    Тест состоит из 12 вопросов 1 части ЕГЭ профильного уровня обучающихся 11 класса

  • Вычисление производной

    03.04.2020 5736

    Данный тест может быть использован в качестве контроля знаний учащихся после изучения темы «Производная»

  • Дифференцирование алгебраической суммы

    18. 04.2020 546 0

    Перед Вами тренировочный тест, проверяющий усвоение небольшой, логически завершенной части темы «Правила дифференцирования». Содержание и уровень сложности включенных в него заданий, в основном, отвечают обязательным требованиям к математической подготовке студентов, обучающихся по специальностям технического профиля. Планируется, что на выполнение этого теста Вы потратите не более 5 минут.

  • Дифференцирование произведения

    18.04.2020 1563 0

    Перед Вами тренировочный тест, проверяющий усвоение небольшой, логически завершенной части темы «Правила дифференцирования». Содержание и уровень сложности включенных в него заданий, в основном, отвечают обязательным требованиям к математической подготовке студентов, обучающихся по специальностям технического профиля. Планируется, что на выполнение этого теста Вы потратите не более 5 минут.

  • Нахождение значений производных в точке

    22.04.2020 244 0

    Тест направлен на формирования навыков поиска производных различных функций и нахождение значений производных

  • Дифференцирование частного

    27.04.2020 1707 0

    Перед Вами тренировочный тест, проверяющий усвоение небольшой, логически завершенной части темы «Правила дифференцирования». Содержание и уровень сложности включенных в него заданий, в основном, отвечают обязательным требованиям к математической подготовке студентов, обучающихся по специальностям технического профиля. Планируется, что на выполнение этого теста Вы потратите не более 5 минут.

  • УД Математика Производная (вариант 2)

    06.05.2020 243 0

    Тест по математике «Вычисление производной» для студентов 1 курса СПО

  • Производная тригонометрической функции

    06.05.2020 1193 0

    Тест предназначен для проверки зананий по теме «Производная тригонометрических функций и их комбинации с элементарными функциями»

  • Нахождение значения производной тригонометрической функции в точке

    08. 05.2020 193 0

    Тест предназначен для проверки зананий по теме «Нахождение значения производной тригонометрической функции в точке»

  • Урок №6 «Признаки возрастания и убывания функции»

    27.05.2020 1893 0

    Данный тест предназначен для закрепления материала по теме «Признак возрастания и убывыания функции». Очень внимательно читайте задание и инструкцию к работе. Желаю удачи!!! 

  • Производная и ее приложения

    29.05.2020 657 0

    Тест предназначен для проверки знания физического и геометрического смысла производной, формул производных элементарных функций, правил вычисления производной, уения применять производную для составления уравнения касательной, исследования функции на монотонность и экстремумы

  • ОУД.

    03 Математика. Итоговое тестирование

    11.06.2020 254 0

    Перед Вами тренировочный тест, проверяющий усвоение небольшой, логически завершенной части учебной дисциплины математика. Содержание и уровень сложности включенных в него заданий, в основном, отвечают обязательным требованиям к математической подготовке студентов, обучающихся по специальностям технического профиля.

  • Производная функции

    06.10.2020 432 0

    Проверка усвоения учащимися темы «Производная функции», умение применять полученные знания на конкретных примерах и задачам физики и геометрии.

  • Применение производной к исследованию функций в заданиях ЕГЭ №1

    27. 12.2020 338 0

    Тест предназначен для обучающихся средней школы  для подготовки к ЕГЭ и  проверки уровня знаний по  теме «Применение производной к исследованию функций и построению графиков».

  • Выполнение задания №7 формата ЕГЭ (профильный уровень)

    09.01.2021 145 0

    В тесте представлены задания на применение геометрического, физического смыслов производной и при исследовании функции

  • Производная. Геометрический смысл производной

    03.03.2021 451 0

    Данный тест предназначен для проверки знаний обучающихся по теме «Производная. Геометрический смысл производной»

  • Производная простейших функций

    04.03.2021 328 0

    Тест по алгебре и началам математического анализа на применение знаний таблицы производных

  • Геометрический смысл производной в задачах ЕГЭ

    30.03.2021 358 0

    Тест  для обучающихся средней школы,  предназначен для подготовки к ЕГЭ и  проверки уровня знаний по  теме «Производная. Геометрический смысл производной».

  • Производные некоторых элементарных функций

    18. 10.2021 623

    Тест по теме «Производные некоторых элементарных функций» предназначен для проверки усвоения указанной темы обучающимися 11 класса

  • Механический смысл производной

    23.10.2021 381 0

    Тест состоит из 7 вопросов по теме «Физический смысл производной», содержит прямые и обратные задания, аналогичные заданию №6 ЕГЭ.

  • Итоговый тест за 1 четверть, 11 класс, математика

    30.10.2021 131 0

    Тест по математике 11 класса за 1 четверть по УМК Алимова Ш. А. (алгебра и начала анализа) и Атанасяна Л. С. (геометрия). Базовый уровень.

  • Производные функции

    05.12.2021 242 0

    Тест для проработки темы -производная. Предназначен для решения и проработки темы учников и их родителей. 

  • Производная сложной функции

    02.02.2022 358 0

    Образовательный тест по теме «Производная сложной функции» позволяет проверить знания школьников или студентов. В тесте требуется как найти сложную функцию из предложенных, так и самостоятельно вычислить производную некоторой сложной функции. Для выполнения заданий необходимо знать таблицу производных и правило вычисления производной сложной функции. Тест одновариантный, состоит из 14 заданий.

  • Правила дифференцирования

    18.03.2022 137 0

    Тест предназначен для проверки знаний по теме «Правила дифференцирования». 

  • Применение производной к исследованию функций и построению графиков.

    14.04.2022 336 0

    Перед Вами тренировочный тест, проверяющий усвоение небольшой, логически завершенной части по теме производная. Содержание и уровень сложности включенных в него заданий, в основном, отвечают обязательным требованиям к математической подготовке студентов, обучающихся по специальностям технического профиля.

  • Тест по теме «Производная»

    23.04.2022 73 0

    Тест по теме «Производная» состоит из четырнадцати вопросов, девять из которых тестовые, остальные с записью ответа

  • Примеры использования производной для нахождения наилучшего решения в прикладных задачах.

    29.04.2022 144 0

    Перед Вами тренировочный тест, проверяющий усвоение небольшой, логически завершенной части темы «Начала математического анализа». Содержание и уровень сложности включенных в него заданий, в основном, отвечают обязательным требованиям к математической подготовке студентов, обучающихся по специальностям технического профиля.

  • Производная. Понятие производной

    02.06.2022 6 0

    Алгебра и начала анализа. Тема «Производная» Тест №1 по теме «Понятие производной. Физический смысл производной»

  • Тест «Правила дифференцирования»

    03.06.2022 69 0

    Тест «Правила дифференцирования». Тест по теме «Правила дифференцирования» проверяет знание  правил дифференцирования — правило дифференцирования суммы двух функций, правило вынесения постоянного множителя за знак производной, правило нахождения производной произведения функций, частного функций,  нахождения производной сложной функции. Тест содержит 10 вопросов. В тесте присутствуют вопросы с единичным верным ответом, вопросы на установление соответствия функции и ее производной и вопрос, в котором нужно записать верный ответ в виде числа.

  • Тест «Производные элементарных функций»

    04.06.2022 41 0

    Тест «Производные элементарных функций». Тест по теме «Производные элементарных функций» проверяет знание формул нахождения производных элементарных функций — степенной, показательной, логарифмической, тригонометрических функций, а также их различных комбинаций и умения вычислять производные функций. Тест содержит 10 вопросов с единичным верным ответом.

  • Тест на соответствие «Производные элементарных функций»

    05. 06.2022 9 0

    Тест на соответствие «Производные элементарных функций». Тест по теме «Производные элементарных функций» проверяет знание формул нахождения производных элементарных функций — степенной, тригонометрических функций, умения вычислять производные функций. Тест содержит 10 функций и 10 производных функций. При выполнении теста необходимо установить соотвтетствие между функцией и ее производной.

  • Тест «Геометрический смысл производной. Уравнение касательной и нормали»

    05.06.2022 5 0

    Тест по теме «Геометрический смысл производной. Уравнение касательной и нормали» проверяет знание геометрического смысла производной,  алгоритм нахождения уравнения касательной и нормали к графику функции в точке; умения составлять уравнение касательной и нормали к графику функции в заданной точке, находить угловой коэффициент касательной к графику функции, угол между касательной к графику и осью Ох. Тест содержит 10 вопросов с выбором единичного и верного ответа, установление верной последовательности и вопросы с вводом верного ответа в виде числа.  

Производная функции y корень из x. Калькулятор онлайн. Найти (с решением) производную функции

Главная > Растения > Производная функции y корень из x. Калькулятор онлайн. Найти (с решением) производную функции

Определение. Пусть функция \(y = f(x) \) определена в некотором интервале, содержащем внутри себя точку \(x_0 \). Дадим аргументу приращение \(\Delta x \) такое, чтобы не выйти из этого интервала. Найдем соответствующее приращение функции \(\Delta y \) (при переходе от точки \(x_0 \) к точке \(x_0 + \Delta x \)) и составим отношение \(\frac{\Delta y}{\Delta x} \). Если существует предел этого отношения при \(\Delta x \rightarrow 0 \), то указанный предел называют производной функции \(y=f(x) \) в точке \(x_0 \) и обозначают \(f»(x_0) \).

$$ \lim_{\Delta x \to 0} \frac{\Delta y}{\Delta x} = f»(x_0) $$

Для обозначения производной часто используют символ y». Отметим, что y» = f(x) — это новая функция, но, естественно, связанная с функцией y = f(x), определенная во всех точках x, в которых существует указанный выше предел. Эту функцию называют так: производная функции у = f(x) .

Геометрический смысл производной состоит в следующем. Если к графику функции у = f(x) в точке с абсциссой х=a можно провести касательную, непараллельную оси y, то f(a) выражает угловой коэффициент касательной:
\(k = f»(a) \)

Поскольку \(k = tg(a) \), то верно равенство \(f»(a) = tg(a) \) .

А теперь истолкуем определение производной с точки зрения приближенных равенств. Пусть функция \(y = f(x) \) имеет производную в конкретной точке \(x \):
$$ \lim_{\Delta x \to 0} \frac{\Delta y}{\Delta x} = f»(x) $$
Это означает, что около точки х выполняется приближенное равенство \(\frac{\Delta y}{\Delta x} \approx f»(x) \), т. 2 \) справедливо приближенное равенство \(\Delta y \approx 2x \cdot \Delta x \). Если внимательно проанализировать определение производной, то мы обнаружим, что в нем заложен алгоритм ее нахождения.

Сформулируем его.

Как найти производную функции у = f(x) ?

1. Зафиксировать значение \(x \), найти \(f(x) \)
2. Дать аргументу \(x \) приращение \(\Delta x \), перейти в новую точку \(x+ \Delta x \), найти \(f(x+ \Delta x) \)
3. Найти приращение функции: \(\Delta y = f(x + \Delta x) — f(x) \)
4. Составить отношение \(\frac{\Delta y}{\Delta x} \)
5. Вычислить $$ \lim_{\Delta x \to 0} \frac{\Delta y}{\Delta x} $$
Этот предел и есть производная функции в точке x.

Если функция у = f(x) имеет производную в точке х, то ее называют дифференцируемой в точке х. Процедуру нахождения производной функции у = f(x) называют дифференцированием функции у = f(x).

Обсудим такой вопрос: как связаны между собой непрерывность и дифференцируемость функции в точке.

Пусть функция у = f(x) дифференцируема в точке х. Тогда к графику функции в точке М(х; f(x)) можно провести касательную, причем, напомним, угловой коэффициент касательной равен f»(x). Такой график не может «разрываться» в точке М, т. е. функция обязана быть непрерывной в точке х.

Это были рассуждения «на пальцах». Приведем более строгое рассуждение. Если функция у = f(x) дифференцируема в точке х, то выполняется приближенное равенство \(\Delta y \approx f»(x) \cdot \Delta x \). Если в этом равенстве \(\Delta x \) устремить к нулю, то и \(\Delta y \) будет стремиться к нулю, а это и есть условие непрерывности функции в точке.

Итак, если функция дифференцируема в точке х, то она и непрерывна в этой точке .

Обратное утверждение неверно. Например: функция у = |х| непрерывна везде, в частности в точке х = 0, но касательная к графику функции в «точке стыка» (0; 0) не существует. Если в некоторой точке к графику функции нельзя провести касательную, то в этой точке не существует производная.

Еще один пример. Функция \(y=\sqrt{x} \) непрерывна на всей числовой прямой, в том числе в точке х = 0. И касательная к графику функции существует в любой точке, в том числе в точке х = 0. Но в этой точке касательная совпадает с осью у, т. е. перпендикулярна оси абсцисс, ее уравнение имеет вид х = 0. Углового коэффициента у такой прямой нет, значит, не существует и \(f»(0) \)

Итак, мы познакомились с новым свойством функции — дифференцируемостью. А как по графику функции можно сделать вывод о ее дифференцируемости?

Ответ фактически получен выше. Если в некоторой точке к графику функции можно провести касательную, не перпендикулярную оси абсцисс, то в этой точке функция дифференцируема. Если в некоторой точке касательная к графику функции не существует или она перпендикулярна оси абсцисс, то в этой точке функция не дифференцируема.

Правила дифференцирования

Операция нахождения производной называется дифференцированием . При выполнении этой операции часто приходится работать с частными, суммами, произведениями функций, а также с «функциями функций», то есть сложными функциями. 2} $$

На этом занятии мы будем учиться применять формулы и правила дифференцирования.

Примеры. Найти производные функций.

1. y=x 7 +x 5 -x 4 +x 3 -x 2 +x-9. Применяем правило I , формулы 4, 2 и 1 . Получаем:

y’=7x 6 +5x 4 -4x 3 +3x 2 -2x+1.

2. y=3x 6 -2x+5. Решаем аналогично, используя те же формулы и формулу 3.

y’=3∙6x 5 -2=18x 5 -2.

Применяем правило I , формулы 3, 5 и 6 и 1.

Применяем правило IV , формулы 5 и 1 .

В пятом примере по правилу I производная суммы равна сумме производных, а производную 1-го слагаемого мы только что находили (пример 4 ), поэтому, будем находить производные 2-го и 3-го слагаемых, а для 1-го слагаемого можем сразу писать результат.

Дифференцируем 2-ое и 3-е слагаемые по формуле 4 . Для этого преобразуем корни третьей и четвертой степеней в знаменателях к степеням с отрицательными показателями, а затем, по 4 формуле, находим производные степеней.

Посмотрите на данный пример и полученный результат. Уловили закономерность? Хорошо. Это означает, что мы получили новую формулу и можем добавить ее в нашу таблицу производных.

Решим шестой пример и выведем еще одну формулу.

Используем правило IV и формулу 4 . Получившиеся дроби сократим.

Смотрим на данную функцию и на ее производную. Вы, конечно, поняли закономерность и готовы назвать формулу:

Учим новые формулы!

Примеры.

1. Найти приращение аргумента и приращение функции y=x 2 , если начальное значение аргумента было равно 4 , а новое —4,01 .

Решение.

Новое значение аргумента х=х 0 +Δx . Подставим данные: 4,01=4+Δх, отсюда приращение аргумента Δх =4,01-4=0,01. Приращение функции, по определению, равно разности между новым и прежним значениями функции, т.е. Δy=f (х 0 +Δх) — f (x 0). Так как у нас функция y=x 2 , то Δу =(х 0 +Δx) 2 — (х 0) 2 =(х 0) 2 +2x 0 · Δx+(Δx) 2 — (х 0) 2 =2x 0 · Δx+(Δx) 2 =

2 · 4 · 0,01+(0,01) 2 =0,08+0,0001=0,0801.

Ответ: приращение аргумента Δх =0,01; приращение функции Δу =0,0801.

Можно было приращение функции найти по-другому: Δy =y (х 0 +Δx) -y (х 0)=у(4,01) -у(4)=4,01 2 -4 2 =16,0801-16=0,0801.

2. Найти угол наклона касательной к графику функции y=f (x) в точке х 0 , если f «(х 0) = 1 .

Решение.

Значение производной в точке касания х 0 и есть значение тангенса угла наклона касательной (геометрический смысл производной). Имеем: f «(х 0) = tgα = 1 → α = 45°, так как tg45°=1.

Ответ: касательная к графику данной функции образует с положительным направлением оси Ох угол, равный 45° .

3. Вывести формулу производной функции y=x n .

Дифференцирование — это действие нахождения производной функции.

При нахождении производных применяют формулы, которые были выведены на основании определения производной, так же, как мы вывели формулу производной степени: (x n)» = nx n-1 .

Вот эти формулы.

Таблицу производных легче будет заучить, проговаривая словесные формулировки:

1. Производная постоянной величины равна нулю.

2. Икс штрих равен единице.

3. Постоянный множитель можно вынести за знак производной.

4. Производная степени равна произведению показателя этой степени на степень с тем же основанием, но показателем на единицу меньше.

5. Производная корня равна единице, деленной на два таких же корня.

6. Производная единицы, деленной на икс равна минус единице, деленной на икс в квадрате.

7. Производная синуса равна косинусу.

8. Производная косинуса равна минус синусу.

9. Производная тангенса равна единице, деленной на квадрат косинуса.

10. Производная котангенса равна минус единице, деленной на квадрат синуса.

Учим правила дифференцирования .

1. Производная алгебраической суммы равна алгебраической сумме производных слагаемых.

2. Производная произведения равна произведению производной первого множителя на второй плюс произведение первого множителя на производную второго.

3. Производная «у», деленного на «вэ» равна дроби, в числителе которой «у штрих умноженный на «вэ» минус «у, умноженный на вэ штрих», а в знаменателе — «вэ в квадрате».

4. Частный случай формулы 3.

Учим вместе!

Страница 1 из 1 1

На котором мы разобрали простейшие производные, а также познакомились с правилами дифференцирования и некоторыми техническими приемами нахождения производных. Таким образом, если с производными функций у Вас не очень или какие-нибудь моменты данной статьи будут не совсем понятны, то сначала ознакомьтесь с вышеуказанным уроком. Пожалуйста, настройтесь на серьезный лад – материал не из простых, но я все-таки постараюсь изложить его просто и доступно.

На практике с производной сложной функции приходится сталкиваться очень часто, я бы даже сказал, почти всегда, когда Вам даны задания на нахождение производных.

Смотрим в таблицу на правило (№5) дифференцирования сложной функции:

Разбираемся. Прежде всего, обратим внимание на запись . Здесь у нас две функции – и , причем функция , образно говоря, вложена в функцию . Функция такого вида (когда одна функция вложена в другую) и называется сложной функцией.

Функцию я буду называть внешней функцией , а функцию – внутренней (или вложенной) функцией .

! Данные определения не являются теоретическими и не должны фигурировать в чистовом оформлении заданий. Я применяю неформальные выражения «внешняя функция», «внутренняя» функция только для того, чтобы Вам легче было понять материал.

Для того, чтобы прояснить ситуацию, рассмотрим:

Пример 1

Найти производную функции

Под синусом у нас находится не просто буква «икс», а целое выражение , поэтому найти производную сразу по таблице не получится. Также мы замечаем, что здесь невозможно применить первые четыре правила, вроде бы есть разность, но дело в том, что «разрывать на части» синус нельзя:

В данном примере уже из моих объяснений интуитивно понятно, что функция – это сложная функция, причем многочлен является внутренней функцией (вложением), а – внешней функцией.

Первый шаг , который нужно выполнить при нахождении производной сложной функции состоит в том, чтобы разобраться, какая функция является внутренней, а какая – внешней .

В случае простых примеров вроде понятно, что под синус вложен многочлен . А как же быть, если всё не очевидно? Как точно определить, какая функция является внешней, а какая внутренней? Для этого я предлагаю использовать следующий прием, который можно проводить мысленно или на черновике.

Представим, что нам нужно вычислить на калькуляторе значение выражения при (вместо единицы может быть любое число).

Что мы вычислим в первую очередь? В первую очередь нужно будет выполнить следующее действие: , поэтому многочлен и будет внутренней функцией :

Во вторую очередь нужно будет найти , поэтому синус – будет внешней функцией:

После того, как мы РАЗОБРАЛИСЬ с внутренней и внешней функциями самое время применить правило дифференцирования сложной функции .

Начинаем решать. Из урока Как найти производную? мы помним, что оформление решения любой производной всегда начинается так – заключаем выражение в скобки и ставим справа вверху штрих:

Сначала находим производную внешней функции (синуса), смотрим на таблицу производных элементарных функций и замечаем, что . Все табличные формулы применимы и в том, случае, если «икс» заменить сложным выражением , в данном случае:

Обратите внимание, что внутренняя функция не изменилась, её мы не трогаем .

Ну и совершенно очевидно, что

Результат применения формулы в чистовом оформлении выглядит так:

Постоянный множитель обычно выносят в начало выражения:

Если осталось какое-либо недопонимание, перепишите решение на бумагу и еще раз прочитайте объяснения.

Пример 2

Найти производную функции

Пример 3

Найти производную функции

Как всегда записываем:

Разбираемся, где у нас внешняя функция, а где внутренняя. Для этого пробуем (мысленно или на черновике) вычислить значение выражения при . Что нужно выполнить в первую очередь? В первую очередь нужно сосчитать чему равно основание: , значит, многочлен – и есть внутренняя функция:

И, только потом выполняется возведение в степень , следовательно, степенная функция – это внешняя функция:

Согласно формуле , сначала нужно найти производную от внешней функции, в данном случае, от степени. Разыскиваем в таблице нужную формулу: . Повторяем еще раз: любая табличная формула справедлива не только для «икс», но и для сложного выражения . Таким образом, результат применения правила дифференцирования сложной функции следующий:

Снова подчеркиваю, что когда мы берем производную от внешней функции , внутренняя функция у нас не меняется:

Теперь осталось найти совсем простую производную от внутренней функции и немного «причесать» результат:

Пример 4

Найти производную функции

Это пример для самостоятельного решения (ответ в конце урока).

Для закрепления понимания производной сложной функции приведу пример без комментариев, попробуйте самостоятельно разобраться, порассуждать, где внешняя и где внутренняя функция, почему задания решены именно так?

Пример 5

а) Найти производную функции

б) Найти производную функции

Пример 6

Найти производную функции

Здесь у нас корень, а для того, чтобы продифференцировать корень, его нужно представить в виде степени . Таким образом, сначала приводим функцию в надлежащий для дифференцирования вид:

Анализируя функцию, приходим к выводу, что сумма трех слагаемых – это внутренняя функция, а возведение в степень – внешняя функция. Применяем правило дифференцирования сложной функции :

Степень снова представляем в виде радикала (корня), а для производной внутренней функции применяем простое правило дифференцирования суммы:

Готово. Можно еще в скобках привести выражение к общему знаменателю и записать всё одной дробью. Красиво, конечно, но когда получаются громоздкие длинные производные – лучше этого не делать (легко запутаться, допустить ненужную ошибку, да и преподавателю будет неудобно проверять).

Пример 7

Найти производную функции

Это пример для самостоятельного решения (ответ в конце урока).

Интересно отметить, что иногда вместо правила дифференцирования сложной функции можно использовать правило дифференцирования частного , но такое решение будет выглядеть как извращение необычно. Вот характерный пример:

Пример 8

Найти производную функции

Здесь можно использовать правило дифференцирования частного , но гораздо выгоднее найти производную через правило дифференцирования сложной функции:

Подготавливаем функцию для дифференцирования – выносим минус за знак производной, а косинус поднимаем в числитель:

Косинус – внутренняя функция, возведение в степень – внешняя функция.
Используем наше правило :

Находим производную внутренней функции, косинус сбрасываем обратно вниз:

Готово. В рассмотренном примере важно не запутаться в знаках. Кстати, попробуйте решить его с помощью правила , ответы должны совпасть.

Пример 9

Найти производную функции

Это пример для самостоятельного решения (ответ в конце урока).

До сих пор мы рассматривали случаи, когда у нас в сложной функции было только одно вложение. В практических же заданиях часто можно встретить производные, где, как матрешки, одна в другую, вложены сразу 3, а то и 4-5 функций.

Пример 10

Найти производную функции

Разбираемся во вложениях этой функции. Пробуем вычислить выражение с помощью подопытного значения . Как бы мы считали на калькуляторе?

Сначала нужно найти , значит, арксинус – самое глубокое вложение:

Затем этот арксинус единицы следует возвести в квадрат :

И, наконец, семерку возводим в степень :

То есть, в данном примере у нас три разные функции и два вложения, при этом, самой внутренней функцией является арксинус, а самой внешней функцией – показательная функция.

Начинаем решать

Согласно правилу сначала нужно взять производную от внешней функции. Смотрим в таблицу производных и находим производную показательной функции: Единственное отличие – вместо «икс» у нас сложное выражение , что не отменяет справедливость данной формулы. Итак, результат применения правила дифференцирования сложной функции следующий.

Здравствуйте, уважаемые читатели. После прочтения статьи у вас, вероятно, возникнет закономерный вопрос: «А зачем, собственно, это надо?». В силу этого сперва считаю необходимым заблаговременно сообщить, что искомый метод решения квадратных уравнений представлен скорее с морально-эстетической стороны математики, нежели со стороны практического сухого применения. Также заранее извиняюсь перед теми читателями, которые посчитают мои дилетантские изречения неприемлемыми. Итак, начнем забивать гвозди микроскопом.

Имеем алгебраическое уравнение второй степени (оно же квадратное) в общем виде:

Перейдем от квадратного уравнения к квадратичной функции:

Где, очевидно, необходимо найти такие значения аргумента функции, в которых оная возвратила бы ноль.

Кажется, нужно просто решить квадратное уравнение с помощью теоремы Виета или через дискриминант . Но мы ведь собрались здесь не для этого. Давайте-ка лучше возьмем производную!

Исходя из определения физического смысла производной первого порядка ясно, что подставляя аргумент в получившуюся выше функцию мы (в частности) получим скорость изменения функции в заданной этим аргументом точке.

На этот раз мы получили «скорость скорости» изменения функции (то бишь ускорение ) в конкретной точке. Немного проанализировав полученное, можно сделать вывод, что «ускорением» является константа, которая не зависит от аргумента функции — запомним это.

Сейчас вспомним немного физику и равноускоренное движение (РУД). Что у нас есть в арсенале? Верно, имеется формула для определения координаты перемещения по оси при искомом движении:

Где — время, — начальная скорость, — ускорение.
Нетрудно заметить, что наша изначальная функция как раз представляет из себя РУД.

Разве формула перемещения для РУД не является следствием решения квадратного уравнения?

Нет. Формула для РУД выше по факту есть результат взятия интеграла от формулы скорости при ПРУД. Или из графика можно найти площадь фигуры. Там вылезет трапеция.
Формула перемещения при РУД не вытекает из решения каких-либо квадратных уравнений. Это очень важно, иначе не было бы смысла статьи.

Теперь осталось разобраться что есть что, и чего нам не хватает.

«Ускорение» у нас уже есть — им является производная второго порядка , выведенная выше. А вот чтобы получить начальную скорость , нам нужно взять в общем-то любой (обозначим его как ) и подставить его в производную теперь уже первого порядка — ибо она и будет искомым.

В таком случае возникает вопрос, какой же нужно взять? Очевидно, такой, чтобы начальная скорость была равна нулю, чтобы формула «перемещения при РУД» стала иметь вид:

В таком случае составим уравнение для поиска :

[подставили в производную первого порядка ]

Корнем такого уравнения относительно будет:

А значением исходной функции при таком аргументе будет:

Теперь становится очевидно, что:

Соединим все «детали пазла» воедино:

Вот мы и получили окончательное решение поставленной задачи. Вообще Америку мы не открыли — мы просто пришли к формуле решения квадратного уравнения через дискриминант окольными путями. Практического смысла это не несет (примерно таким же образом можно решать уравнения первой/второй степени любого (не обязательно общего) вида).

Целью этой статьи является, в частности, подогрев интереса к анализу мат. функций и вообще к математике.

С вами был Петр, спасибо за внимание!

Решать физические задачи или примеры по математике совершенно невозможно без знаний о производной и методах ее вычисления. Производная — одно из важнейших понятий математического анализа. Этой фундаментальной теме мы и решили посвятить сегодняшнюю статью. Что такое производная, каков ее физический и геометрический смысл, как посчитать производную функции? Все эти вопросы можно объединить в один: как понять производную?

Геометрический и физический смысл производной

Пусть есть функция f(x) , заданная в некотором интервале (a, b) . Точки х и х0 принадлежат этому интервалу. При изменении х меняется и сама функция. Изменение аргумента – разность его значений х-х0 . Эта разность записывается как дельта икс и называется приращением аргумента. Изменением или приращением функции называется разность значений функции в двух точках. Определение производной:

Производная функции в точке – предел отношения приращения функции в данной точке к приращению аргумента, когда последнее стремится к нулю.

Иначе это можно записать так:

Какой смысл в нахождении такого предела? А вот какой:

производная от функции в точке равна тангенсу угла между осью OX и касательной к графику функции в данной точке.


Физический смысл производной: производная пути по времени равна скорости прямолинейного движения.

Действительно, еще со школьных времен всем известно, что скорость – это частное пути x=f(t) и времени t . Средняя скорость за некоторый промежуток времени:

Чтобы узнать скорость движения в момент времени t0 нужно вычислить предел:

Правило первое: выносим константу

Константу можно вынести за знак производной. Более того — это нужно делать. При решении примеров по математике возьмите за правило — если можете упростить выражение, обязательно упрощайте .

Пример. Вычислим производную:

Правило второе: производная суммы функций

Производная суммы двух функций равна сумме производных этих функций. То же самое справедливо и для производной разности функций.

Не будем приводить доказательство этой теоремы, а лучше рассмотрим практический пример.

Найти производную функции:

Правило третье: производная произведения функций

Производная произведения двух дифференцируемых функций вычисляется по формуле:

Пример: найти производную функции:

Решение:

Здесь важно сказать о вычислении производных сложных функций. Производная сложной функции равна произведению производной этой функции по промежуточному аргументу на производную промежуточного аргумента по независимой переменной.

В вышеуказанном примере мы встречаем выражение:

В данном случае промежуточный аргумент – 8х в пятой степени. Для того, чтобы вычислить производную такого выражения сначала считаем производную внешней функции по промежуточному аргументу, а потом умножаем на производную непосредственно самого промежуточного аргумента по независимой переменной.

Правило четвертое: производная частного двух функций

Формула для определения производной от частного двух функций:

Мы постарались рассказать о производных для чайников с нуля. Эта тема не так проста, как кажется, поэтому предупреждаем: в примерах часто встречаются ловушки, так что будьте внимательны при вычислении производных.

С любым вопросом по этой и другим темам вы можете обратиться в студенческий сервис . За короткий срок мы поможем решить самую сложную контрольную и разобраться с заданиями, даже если вы никогда раньше не занимались вычислением производных.

Производная функции — презентация онлайн

Похожие презентации:

Производная функции

Функции и их свойства. Предел последовательности и функции. Производная функции и дифференциал

Производная функции

Производная сложной функции

Понятие производной

Производная сложной функции

Производная функции

Производная функции

Производная функции

Производная сложной функции

Математика
группа 201
Производная
функции
Преподаватель Морозова И.М.

2. Домашнее задание

1. Изучить презентацию.
2. Выписать в тетрадь примеры решения задач из данной
презентации.
3. Выполнить тест в тетради в конце презентации. Фото теста
прислать на почту [email protected] Вариант – номер вопросаномер ответа. В письме написать ФИО и номер группы.
4. Все вопросы по теме и выполнению данных заданий можно
задавать в WhatsApp или Viber по номеру 8-912-61-85-301.
Как найти путь, который прошла точка?
x0
x
x-x0=Δx
путь
изменение
координаты
приращение
аргумента
y
y= f (x)
y
Δx=x-x0 приращение
аргумента
Δy=y-y0 приращение
функции
Δy=Δf
y0
0
Δx
x0
x=x0+Δx
x
y
Δy=y(x0+Δx)-y(x0)
y= f (x)
y(x0+Δx)
Δy=Δf
y(x0)
0
Δx→0
x0
x
x0+Δx
Определение
Производной функции называется предел отношения
приращения функции к приращению аргумента, когда
приращение аргумента стремится к нулю.
y ( x0 x) y ( x0 )
y
y lim
lim
x 0 x
x 0
x
Обозначение
y
или
f (x )
Схема вычисления производной функции
1. Найти приращение функции на отрезке [ x; x+Δx]:
y y ( x x) y ( x)
2. Разделить приращение функции на приращение
аргумента: y
y ( x x) y ( x)
x
x
3. Найти предел отношения приращения функции к
приращению аргумента, когда приращение аргумента
стремится к нулю.
y
y ( x x) y ( x)
y lim
lim
x 0 x
x 0
x
Пример: Вычислить производную функции y=x2
Решение: Используем схему вычисления производной по
действиям:
1.
y y ( x x) y ( x) ( x x) x
2
2
x 2 2 x x x 2 x 2 2 x x x 2
2.
3.
y 2 x x x
x (2 x x)
x
x
x
2
2x x
y
y lim
lim 2 x x
x 0 x
x 0
lim 2 x lim x 2x 0 2x
x 0
x 0
Задание: Найти производную функции:
1.
y x
2.
y C , где С число
Решение 2
3.
y kx b, где k и b числа
Решение 3
4.
5.
3
Решение 1
y ax , где a число
2
1
y
x
Решение 5
6.
Решение 4
y x
Решение 6
y (функция ) y (производная функции )
y x
3
y C , где С число
3x
2
0
y kx b, где k и b числа
k
y ax , где a число
2ax
2
1
y
x
y x
1
x2
1
2 x
Решение 1: Вычислить производную функции y x 3
1.
y y ( x x) y ( x) ( x x) x
3
3
x 3 3x 2 x 3x x 2 x 3 x 3
3x 2 x 3x x 2 x 3
2.
3.
y 3x 2 x 3x x 2 x 3
3x 2 3x x x 2
x
x
y
2
2
y lim
lim 3x 3 x x x
x 0 x
x 0
2
2
lim 3x lim 3x x lim x
x 0
2
x 0
3x 0 0 3x
x 0
2
Задания
Решение 2: Вычислить производную функции y C , где С число
1.
2.
3.
y y ( x x) y ( x) С С 0
y
0
0
x x
y
y lim
lim 0
x 0 x
x 0
0
Задания
Решение 3: Вычислить производную функции y kx b, где k и b числа
1.
y y ( x x) y ( x) k x x b kx b
kx k x b kx b k x
2.
3.
y k x
x
x
k
y
y lim
lim k
x 0 x
x 0
k
Задания
Решение 4: Вычислить производную функции
1.
y y ( x x) y ( x)
y ax 2 , где a число
a( x x) ax
2
2
a( x 2 2 x x x 2 ) ax 2
ax 2 2ax x a x 2 ax 2 2ax x a x 2
2.
y 2ax x a x 2
x (2ax a x)
x
x
x
2ax a x
3.
y
y lim
lim (2ax a x)
x 0 x
x 0
lim 2ax lim a x 2ax 0 2ax
x 0
x 0
Задания
1
y
Решение 5: Вычислить производную функции
x
1
1
y y ( x x) y ( x)
x x x
1 x
1 ( x x) x x x x
x( x x) x( x x)
x( x x) x( x x)
1.
2.
3.
x
y
1
x
x( x x)
2
x
x x x
x( x x) x
x
y
1
1
1
y lim
lim ( 2
) 2
2
x 0 x
x 0 x x x
x 0 x
Задания
y x
Решение 6: Вычислить производную функции
1.
y y ( x x) y ( x)
2.
y
x
x x x
x
x x x
x x x x x x
x x x x
x x x
x
1
x x x x x x x x x x x
3.
y
1
y lim
lim (
)
x 0 x
x 0
x x x
1
1
x 0 x 2 x
Задания
ПРОИЗВОДНАЯ СЛОЖНОЙ ФУНКЦИИ
Сложная функция: y g f x .
1) y 3 x 2 x .
Примеры:
5
2
2) y sin x . y f ;
f sin x.
y f 5;
f 3 x 2 2 x.
3) y cos 2 x .
3
y cos f ;
f 2x .
3
Правило нахождения производной сложной функции
g
/
f x g f f x
/
/
(производная сложной функции равна
производной основной функции
на производную внутренней функции)
ПРОИЗВОДНАЯ СЛОЖНОЙ ФУНКЦИИ
Сложная функция: y g f x .
Правило нахождения производной сложной функции
g / f x g / f f / x
Простая
функция
Производная
простой
функции
xn
nx
1
x
1
2
x
x
2 x
sin x
cos x
n 1
1
(производная сложной функции равна
производной основной функции
на производную внутренней функции)
Сложная Производная сложной функции
функция
f n x
1
f x
f x
sin f x
n f n 1 x f / x
f / x
f 2 x
f / x
2 f x
cos f x f / x
1)y = x•cos x
Найти у´
y´=(x•cos x)´= x´•cos x + x•(cos x)´=1•cos x + x•(-sin x)= cos x — x•sin x
2)y = x5+sin x
Найти у´
y´=(x5+sin x)´= (x5)´+(sin x)´= 5×4+cosx
3) y = x•sin x
Найти у´
y´=(x•sin x)´= x´•sin x + x•(sin x)´=1•sin + x•cos x= sin x + x•cos x
4) y = 4 x +tg x
Найти у´
y´=(4 x +tg x)´= (4 x )´+(tg x)´= 4
5) y = sin x –2х
1
2 х
Найти у´(0)
y´=(sin x — 2x)´= (sin x)´-(2x)´= cosx — 2
у´(0) = cos 0 — 2 = 1-2 = -1
+
1
cos 2 x
=
2
х
+
1
cos 2 x
Производная сложной
функции
Функция h есть
сложная функция, составленная
из функций g и f, если
h(x)=g(f(x))
f(x) – «внутренняя функция»
g(f) – «внешняя функция»
Определим внутреннюю(f) и внешнюю(g)
элементарные функции, из которых составлена
сложная функция h(x)=g(f(x))
h(x) = cos3x
f(x) = 3x
g(f) = cosf
1)
h(x) = tg(2x- /4)
f(x)= 2x- /4
g(f) = tgf
2)
h(x)=(3-5x)5
f(x) = 3-5x
g(f) = f 5
4) h(x) = sin x
f(x) = sin x
g(f) = f
3)
Определите внутреннюю(f) и внешнюю(g)
элементарные функции, из которых составлена
сложная функция y=g(f(x))
1) y = 9-x2
2) y = sin
x
3
3) y = 2(3×3-6x)7
f(x)= 9-x2,
f(x)=
x
3
,
f(x)= 3×3-6x,
g(f)= f
g(f)=sin f
g(f)=2f 7
Формула производной сложной
функции
.
h΄(x) = g´(f) f ´(x)
h΄(x) = g´(f) • f ´(x)
Алгоритм нахождения производной
сложной функции
1)
Определи внутреннюю и внешнюю элементарные
функции f(x) и g(f)
2)
Найди производную внутренней функции f
3)
Найди производную внешней функции g´(f)
4)
´(x)
Перемножь производные внутренней и внешней
функции и получишь производную сложной
функции
h΄(x) = f ´(x) • g´(f)

25. Производная сложной функции

Задание 1. Найдите производную функции
h(x) = (2x+3)100
1. Определим внутреннюю(f)и внешнюю(g) функции
f(x)=2x+3
g(f)=f 100
2. Найдем производную внутренней функции
f ´(x)=(2x+3)´=2
3. Найдем производную внешней функции
g´(f)=(f 100)´=100 f 99
4. Перемножим производные внутренней и внешней
функций
h´ (x) = 2 . 100 f 99= 200 f 99 = 200(2x+3) 99

26. Определим внутреннюю(f) и внешнюю(g) элементарные функции, из которых составлена сложная функция h(x)=g(f(x))

Задание 2. Найдите производную функции
y(x) =4cos 3x
1. Определим внутреннюю(f)и внешнюю(g) функции
f(x)=3x
g(f)=4cosf
2. Найдем производную внутренней функции
f ´(x)=(3x)´=3
3. Найдем производную внешней функции
g´(f)=(4cosf)´= — 4sin f
4. Перемножим производные внутренней и внешней
функций
y´ (x) = 3 .( — 4sin f) = — 12sin f = — 12sin3x
Задание 3. Найдите производную функции
a) y= 9 х
2
a) f(x)=9-x2, g(f)= f
f ´(x)=(9-x2)´= -2x
g´ (f)=( f )´=
1
х
б) у= 6sin 3
х
б) f(x)=
, g(f)= 6sin f
3
f ´(x)=( х ) ´= 1
3
3
2 f
g´ (f)=(6sin f)´=6cos f
2x
y ´ = -2x
==
2 f
2 f
1
y´ = .6cos f = 2cos f=
3
x
=f
х
= 2cos
3
1
=
x
9 x2
Тест
1 вариант
А
В
С
1.
2.
3
4.
5.
2 вариант
1
2
3
4
5
А
В
С
Многие, которым никогда не представлялось
случая более узнать математику, смешивают
ее с арифметикой и считают ее наукой сухой.
В сущности же эта наука, требующая
наиболее фантазии, и один из первых
математиков нашего века говорит совершенно
верно, что нельзя быть математиком, не
будучи в то же время поэтом в душе.
С. КОВАЛЕВСКАЯ
Благодарю за внимание!
Учитесь с удовольствием!

English     Русский Правила

Как найти производную функции с примерами решения и готовыми ответами

Содержание:

  1. Определение производной
  2. Вычисление производной
  3. Основные правила дифференцирования
  4. Таблица производных основных элементарных и сложных функций
  5. Примеры с решением
  6. Определение производной функции, ее физический и геометрический смысл 

Определение производной

Предел отношения приращения функции к приращению аргумента, если приращение аргумента стремится к нулю и если этот предел существует, называется производной функции в точке (рис. 117).

.

Нахождение производной функции называется дифференцированием. Функция, дифференцируемая в каждой точке некоторого интервала, называется дифференцируемой на этом интервале.

Рис. 117

По этой ссылке вы найдёте полный курс лекций по высшей математике:

Высшая математика: лекции, формулы, теоремы, примеры задач с решением

Вычисление производной

Основные правила дифференцирования

— постоянный множитель можно выносить за знак производной.

— производная суммы функций равна сумме производных этих функций.

Производная произведения двух функций вычисляется по формуле .

Производная отношения двух функций вычисляется по формуле .

Таблица производных основных элементарных и сложных функций

Уравнение касательной к графику функции в точке :

.

Возможно вам будут полезны данные страницы:

Найти область определения функции

Формула Байеса

Формулы производных

Таблица производных

Примеры с решением

Пример 1.

Найдите производную функции

.

Решение:

Вынесем постоянный множитель за знак производной.

Применим формулу . Здесь .

.

Ответ: .

Пример 2.

Решите уравнение

.

Решение:

Область определения функции .

.

Уравнение имеет вид .

Ответ: 0.

Пример 3.

.

Решение:

Область определения функции: .

.

Уравнение имеет вид области определения

Ответ: 2.

Пример 4.

Решение:

Воспользуемся формулой

уравнение:

Ответ: -2.

Запомните!

Производная функции в точке равна угловому коэффициенту касательной, проведенной к графику функции в точке с абсциссой .

Угловой коэффициент прямой равен тангенсу угла, который эта прямая образует с положительным направлением оси абсцисс (рис. 118, 119).

В уравнении прямой линии число — угловой коэффициент.

Рис. 118 Рис. 119

Определение производной функции, ее физический и геометрический смысл 

Пусть дана функция . Фиксируем произвольную точку из области определения функции . Пусть — другая точка из , тогда разность называется приращением аргумента и обозначается а разность значений функции называется приращением функции и обозначается

Производной функции в точке называется предел отношения ее приращения в этой точке к приращению аргумента при (если этот предел существует). Производная обозначается или . Процесс нахождения производной называется дифференцированием. Итак, . В физическом смысле отношение является средней скоростью изменения функции на отрезке — Мгновенной скоростью изменения этой функции в точке .

Например, если функция задает зависимость пути , пройденного некоторым телом, от времени , то производная является скоростью движения; если функция дает зависимость количества производимой кем-то продукции от времени, то ее производная является производительностью труда.

Рис. 1

Геометрически отношение является тангенсом угла (рис. 1), который образует секущая, проходящая через точки и с осью . Если то точка стремится к точке , а угол стремится к углу который образует касательная к графику функции в точке с осью и, следовательно, . Отсюда получаем (геометрический смысл производной), где — угловой коэффициент касательной к графику функции в точке . Следовательно, уравнение касательной к графику функции в точке есть .

Функция называется дифференцируемой в точке , если ее приращение в этой точке можно представить в виде

(1)

где — константа, а — бесконечно малая функция (б.м.ф.) при .

Теорема 1. Необходимым и достаточным условием дифференцирования функции в некоторой точке является существование ее производной в этой точке.

Доказательство необходимости. Пусть функция дифференцируема в точке . Тогда , где — б.м.ф. при . . Отсюда по обратной лемме (см. п. 3, раздел 4.3) , т.е. производная функции существует.

Доказательство достаточности. Пусть существует производная функции . Согласно лемме (см. также п. 3 раздела 4,3) , где — б.м.ф. при , т.е. функция дифференцируема в точке .

Замечание. В ходе доказательства также получено, что для дифференцируемой функции равенство (1) можно записывать в виде , где — б.м.ф, при .

Теорема 2. Если функция дифференцируема в точке, то она непрерывна в ней.

Доказательство. Если функция дифференцируема в точке , то , где — б.м.ф. при . Откуда , т.е. функция непрерывна в точке .

Правила дифференцирования (нахождения производных функций). Производные основных элементарных функций:

  1. постоянная величина: ;
  2. степенная функция: ;
  3. показательная функция: ;
  4. показательная функция с основанием ;
  5. логарифмическая функция с натуральным логарифмом: ;
  6. логарифмическая функция: ;
  7. тригонометрические функции: .

Структурные правила дифференцирования:

  1. производная суммы: ;
  2. производная произведения: ;
  3. производная частного: ;
  4. производная сложной функции: ;
  5. производная обратной функции: . Вывод этих правил можно посмотреть в учебниках.

Производные неявных и параметрических функций. Пусть функция задана неявно при помощи уравнения . Чтобы найти её производную, нужно это равенство продифференцировать по с учетом зависимости от и затем решить полученное уравнение относительно .

Пример 5.

Найти , от неявной функции .

Решение:

Продифференцируем данное равенство по , затем найдем откуда .

Функция называется параметрической, если значения функции и значения аргумента задаются при помощи функций от вспомогательной величины , называемой параметром: и .

Правило нахождения производной параметрической функции: если существуют .

Пример 6.

Найти от функции , заданной параметрически: .

Решение:

, значит, .

Производные высших порядков. Производная функции является новой функцией того же аргумента . Производная этой функции называется производной 2-го порядка по отношению к исходной функции и она обозначается или . Аналогично определяются производная 3-го порядка, производная 4-го порядка и дальнейшие производные.

Пример 7.

Функция в точке 0 не имеет производной. Надо заменить ее параболой на некотором интервале, содержащем нуль, чтобы получившаяся функция всюду имела производную, как доказано на рис. 2. У Крылова в качестве функции было сопряжение круга с прямой.

Рис. 2

Решение:

Будем искать параболу в виде . Тогда в точке сопряжения параболы и правой половины графика выполнены уравнения (точка принадлежит обоим графикам и в этой точке равны производные):

Решая систему, получаем: , откуда искомая функция есть

Замечание.

В последние годы большую популярность в теории приближений в некоторых прикладных исследованиях приобрели сплайны.

Так называются функции, представленные на разных участках многочленами и имеющие непрерывные производные до какого-то порядка включительно. Крылов и мы, при решении задачи Крылова, как раз построили сплайн — составленную из параболы и двух прямых лучей функцию, имеющую непрерывную 1-ю производную (у Крылова сплайн имел непрерывную 2-ю производную).

Определение производной функции в точке

Определение производной

Производная функции в точке
Пусть функция f(x) определена в некоторой окрестности точки x.
Производной f′(x) функции f(x) в точке x называется конечный предел отношения приращения функции к приращению ее аргумента, когда последний стремится к нулю:
(1)   .
Приращение аргумента функции
в точке x – это разность значений аргумента в некоторой точке и точке x:
.
Приращение аргумента является независимой переменной.
Приращение функции
в точке x – это разность значений функции в некоторой точке и точке x: .
Приращение функции является зависимой переменной. Оно зависит от и , или от и .
Дифференцирование
– это процесс вычисления производной.

В определении (1), приращение аргумента является одной переменной, хотя ее обозначение состоит из двух букв: и . Обычно переменную принято обозначать одной буквой или буквой с одним или несколькими индексами. Но приращение в математическом анализе настолько часто встречается, что его обозначают с небольшим нарушением правил. Приращение функции также является одной переменной.

В приведенных выше определениях, является независимой переменной, а – зависимой. зависит от двух переменных. Если использовать выражение , то приращение функции зависит от переменных x и . Если использовать , то зависит от x и . Но когда мы вычисляем производную в заданной точке x, то считаем, что x является постоянной. Тогда является функцией, зависящей только от одной переменной . Таким образом задача о нахождении производной в точке x сводится к задаче о вычислении предела от функции , зависящей от одной переменной при , или от функции , зависящей от одной переменной при .

В правой части (1) мы сделали замену, и перешли от переменной к переменной . Тогда . При ,
.

После того, как мы нашли производную в заданной точке, то x уже можно считать не фиксированным числом, а переменной. То есть предел (1) можно рассматривать как функцию от x.

Еще раз подчеркнем, что выражение является функцией от двух переменных: x и . А выражение , полученное после вычисления предела, зависит только от одной переменной x.

Ниже приводятся примеры вычислений производных ⇓, используя определение ⇑.

Обозначение производной

Обозначение Лагранжа

Наиболее популярным является обозначение Лагранжа. Производную функции обозначают как и саму функцию, добавляя штрих после ее характеристики: . Если функция задана алгебраическим выражением, то это выражение заключают в скобки, и ставят знак штриха справа за закрывающей скобкой: . При этом производная также является функцией от той же переменной x, что и . Правда область определения производной может не совпадать с областью определения функции, а является ее подмножеством.

Напомним, что в обозначении функции фигурируют три символа: независимая переменная, характеристика функции и зависимая переменная (см. «Определение функции»). Так, в выражении
(2)   ,
x является независимой переменной, или аргументом функции; f   – характеристикой функции; y   – зависимой переменной, или значением функции. Обозначение зависимой переменной может совпадать или не совпадать с обозначением характеристики. Производную функции (2) обозначают так:
.

Независимую переменную производной обозначают так же, как и независимую переменную функции. В нашем случае это x.

Характеристику производной обозначают тем же символом, что и характеристику функции, добавляя штрих справа: .

Зависимую переменную производной обозначают аналогично характеристике, добавляя штрих к обозначению зависимой переменной функции. Так, для примера (2), это будет : .

Если функция зависит от нескольких переменных, например
(3)   ,
но все кроме одной считают постоянными, то к характеристике производной добавляют нижний индекс, обозначающий ту переменную, по которой вычисляют производную. При этом знак штриха может быть опущен. Например, следующие два обозначения эквивалентны: . Здесь подразумевается, что переменные и мы считаем постоянными. Тогда в данном контексте, является функцией от одной переменной . К зависимой переменной производной также добавляют нижний индекс переменной, по которой выполняется дифференцирование:
.
Подобные производные функций от нескольких переменных называются частными производными. Детально они будут рассмотрены позже.

Нижний индекс добавляют и при вычислениях, связанных со сложными функциями. Пусть, например, функцию можно представить как сложную: , составленную из двух функций: и . При этом множества значений функций и совпадают. Поэтому их удобно обозначить одной переменной y. Тогда производную от y, выраженную через переменную x, обозначают как :
.
А производную от y, выраженную через переменную , обозначают как :
.

Обозначение производной по времени в физике

В механике и физике, производную по времени обозначают не штрихом, а точкой над зависимой переменной. Обычно время обозначают буквой t. Тогда
.

Обозначение Лейбница

В способе Лейбница, зависимую переменную обозначают в форме отношения дифференциалов:
.
Этот способ удобен, поскольку указывает, по какой переменной ведется дифференцирование. Такой способ применяется только для функций от одной переменной. Для функций от многих переменных используют обозначение частной производной: .

Иногда в форме дифференциалов обозначают характеристику производной, добавляя справа аргумент:
.
Однако этот способ скорее неудачен, и может привести к путанице.

Обозначение Коши

Также, для обозначения производной, используют обозначение Коши:
.
Но мы не будем им пользоваться.

Существование производной

Рассмотрим вопрос о существовании предела, который используется при вычислении производной, при заданном значении x:
(4)   .
Здесь могут возникнуть три случая:
1) в точке x существует конечный предел (4);
2) существует бесконечный предел или ;
3) предела (4) не существует.

1) Если существует конечный предел (4), то говорят, что функция имеет производную в точке x.

2) Если в некоторой точке x существует бесконечный предел (4), то говорят, что производной в этой точке не существует. Это согласуется с определением, ⇑ в котором указано, что производной называется конечный предел. Однако при этом говорят, что функция f имеет в точке x бесконечную производную, равную или . Здесь стоит обратить внимание на различие в определении предела и производной. Возможна ситуация, когда предел (4) существует (равный бесконечности), но при этом производная не существует (хотя существует ее значение, равное бесконечности).
См. пример ⇓.

3) Если предела (4) не существует, то функция не имеет производной в точке x.

Производные справа и слева

Определение

Правая (левая) производная функции f в точке x
Пусть функция f(x) определена в правой окрестности точки x. Тогда правой производной функции f в точке x называется правый предел
.
Соответственно, если функция определена в левой окрестности x, то левой производной функции f в точке x называется левый предел
.
Правую (левую) производную также называют производной справа (слева) в точке x, или правосторонней (левосторонней) производной в точке x.

Лемма об односторонних производных

Функция имеет в точке x производную тогда и только тогда, когда
она имеет в этой точке производные справа и слева, и они равны:
.
При этом
.

Доказательство

Для доказательства применим теорему об односторонних пределах.

Пусть существует производная функции в точке x. Это означает, что она определена в некоторой окрестности точки x, и существует конечный предел функции при :
.
Но тогда существуют правая и левая окрестности точки x, на которых определена. По теореме об односторонних пределах, существуют равные правый и левый пределы:
.
Отсюда следует, что в точке x существуют односторонние производные
.

Пусть теперь, в точке x, существуют равные односторонние производные:
.
Это означает, что существуют правая и левая окрестности точки x, в которой определена . И существуют односторонние равные пределы:
.
Отсюда следует, что существует двусторонняя окрестность точки x, на которой определена . И по теореме об односторонних пределах, существует двусторонний предел:
.
Это означает, что в точке x существует производная
.

Лемма доказана.

Следствие о неравных односторонних производных
Если функция имеет в точке x не равные односторонние производные:
,
то она не имеет производной в этой точке.

Действительно, допустим противное. Пусть функция имеет в точке x не равные односторонние производные, но при этом имеет производную в этой точке. Тогда, согласно лемме об односторонних производных, она имеет в этой точке равные производные слева и справа, что противоречит предположению.

См. пример ⇓.

Примеры вычисления производной, используя определение

Все примеры Здесь и далее мы приводим подробные решения примеров, в которых нужно вычислить производную функции , используя определение производной ⇑.
  решение ⇓ ;    ⇓ ;    ⇓ .

Пример

Все примеры ⇑ Найти производную функции , используя определение производной.

Решение

Функция определена для всех x. Поэтому она определена в любой окрестности любой точки x. Используем определение (1). Считаем, что x – фиксированное число, то есть что его значение задано. Найдем приращение функции в точке x:

.
Находим отношение приращения функции к приращению ее аргумента:
.
Находим предел функции , зависящей от переменной . При этом считаем, что x является фиксированным, заданным числом:
.

Итак, мы нашли производную:
.
Поскольку вычисленный нами предел существует, и является конечным числом для всех x, то функция имеет производную для всех значений аргумента x.

Ответ

.

Пример бесконечной производной +∞

Все примеры ⇑ Найдем производную функции .

Решение


Производная функции в точке x = 0 равна плюс бесконечности.

Функция определена для всех x. Найдем отношение приращения функции к приращению ее аргумента в точке x:
.
Применим формулу . Тогда
;
(5)   .
Считаем, что x является фиксированным числом. Тогда отношение является функцией от одной переменной : . При она определена для всех . При она определена для всех .

Пусть . Тогда:
.
Пусть . Подставим в (5) :
.
Поскольку , то
.

Ответ

Таким образом мы нашли, что функция имеет производную для всех . При функция не имеет производной, она равна .

Пример

Все примеры ⇑ Найдем производную функции . Покажем, что несмотря на то, что функция определена для всех x, ее производная в точке не существует.

Решение


Функция y = |x| не имеет производной в точке x = 0.

Функция определена для всех значений аргумента x. Поэтому она определена в любой окрестности произвольной точки x.

1. Пусть . Тогда ,
.

2. Пусть . Тогда ,

.

3. Рассмотрим точку . В ней
.
Найдем производную справа в точке . При этом ,
.
Теперь найдем производную слева в точке . В этом случае ,
.

Итак, мы нашли, что односторонние производные в точке существуют, но они не равны друг другу:
.
Согласно следствию леммы об односторонних производных, производной функции в точке не существует.

Ответ

;
;
.
В точке производная не существует.

Использованная литература:
Г.Е. Иванов. Лекции по математическому анализу. Часть 1. Москва, МФТИ, 2018.
Л.Д. Кудрявцев. Курс математического анализа. Том 1. Москва, 2003.
С.М. Никольский. Курс математического анализа. Том 1. Москва, 1983.

▷Step by Step Приложения для TI-Nspire CX и CX CAS Скачать бесплатно. Пройдите курсы по математике, естественным наукам и бизнесу



Подготовка к экзаменам по математике и естественным наукам, домашнее задание. Проверьте свою работу.


— Шаг за шагом к успеху. Приложения запускаются за считанные минуты.
Сначала протестируйте наши бесплатные пробные версии.—
95% купили больше ПРИЛОЖЕНИЙ. 97% сообщили об улучшении показателей. Рейтинг: 4,89 из 5 звезд. Доступно 46 ПРИЛОЖЕНИЙ.

КОВИД СПЕЦИАЛЬНЫЙ

Купите 3 приложения Made Easy по цене 2 приложений.

Выберите 3 приложения. EasyBusiness Stats Made EasyCalculus with Physics Apps Calculo de Manera Facil Chemistry Made EasyChemie Leicht GemachtQuimica de Manera FacilCollege Algebra Made Easy CX CASCollege Algebra Made Easy CXComplex Analysis Made EasyConics Made EasyConico de Manera FacilDifferential Equations Made EasyEcuaciones Diferencial de Manera FacilDifferential Gleichungen Leicht Gemacht DiscreteMDisateM de Manera FacilEconomics Made EasyEinheiten Umwandler mit SchrittenElectrical Engineering Made EasyElectronik Leicht GemachtEngineering Economics Made EasyEngineering Mathematics Made EasyIngenie ur Mathematik Leicht GemachtFinance Made EasyКонечная математика Made EasyGeometry Made EasyGeometrie Leicht GmachtGeometria de Manera FacilLand Survey Made EasyLinear Algebra Made EasyLinear Algebra de Manera FacilLineare Algebra Leicht GmachtMathematical Economics Made EasyMathematical Economics Made EasyMathematical Economics Made EasyMathematical Economics Made EasyMathematical Economics Made EasyCXMatrix Made EasyЧисленный анализ Made EasyPhysik Leicht EasyPhysik Research Made EasyPhysik Research Made Easy GemachtFisica de Manera FacilPortfolio & Stocks Made EasyPreCalculus Made EasyPreCalculus Made Easy CXPreCalculus Made Easy CXPreCalculo de Manera FacilReal Estate Made EasySAT Made EasySAT Subject Test MathSignals and Systems Made EasyStatistics and Probability Made EasyStatistik Leicht GemachtEstadisticas de Manera FacilStatic and Dynamics Made EasyStatik und Dynamik Leicht GemachtStep by Step Equation Solver Ecuaciones de Manera FacilПошаговый конвертер единиц измеренияThermodynamics Made EasyThermodynamik Leicht GemachtTrigonometry Made EasyTrigonometria de Manera FacilTr igonometrie Leicht GemachtVector Calculus Made EasyVektor Analysis Leicht GemachtWirtschaftsmathematik Leicht Gemacht

Calculus Made EasyACT Made EasyAccounting Made EasyAerodynamics Made EasyAnalysis Leicht GemachtAnalysis mit PhysikAlgebra Made Easy CX CASAlgebra Made Easy CXAlgebra Leicht Gemacht CX CASAlgebra de Manera FacilAlgebra de Manera Facil CXApplications and Optimizations Made EasyBiology Made EasyBiostatistics AppBusiness Calculus Made EasyBusiness Stats with Physics with de Manera Facil Chemistry Made EasyChemie Leicht GemachtQuimica de Manera FacilCollege Algebra Made Easy CX CASCollege Algebra Made Easy CXComplex Analysis Made EasyConics Made EasyConico de Manera FacilDifferential Equations Made EasyEcuaciones Diferencial de Manera FacilDifferential Gleichungen Leicht GemachtDiscrete Math Made EasyMatematicas Discretas de Manera FacilEconomics Made EasyEinheiten Umwandler mit SchrittenElectrical Engineering Made EasyElectronik Leicht GemachtEngineering Economics Made EasyEngineering Mathematics Made EasyIngenieur Mathematik Leicht GemachtFinance Mad e EasyFinite Math Made EasyGeometry Made EasyGeometrie Leicht GemachtGeometria de Manera FacilLand Surveying Made EasyLinear Algebra Made EasyLinear Algebra de Manera FacilLineare Algebra Leicht GemachtMathematical Economics Made EasyMathematical Economics Made EasyMathematical Economics Made EasyCXMatrix Made EasyNumerical Analysis Made EasyNumber Theory Made EasyProperties Research Made EasyPhysik Made EasyPhysik Leicht Gemacht & Stocks Made EasyPreCalculus Made EasyPreCalculus Made Easy CXPreCalculus Made Easy CXPreCalculo de Manera FacilReal Estate Made EasySAT Made EasySAT Subject Test MathСигналы и системы Made EasyСтатистика и вероятность Made EasyStatistik Leicht GemachtEstadisticas de Manera FacilStatik und Dynamics Made EasyStatik und Dynamik Leicht GemachtStep by Step Equal SolverSolucionador de EcuacionesStep de Manera Facil by Step Unit ConverterThermodynamic Made EasyThermodynamik Leicht GemachtТригонометрия Made EasyTrigonometria de Manera FacilTrigonometrie Leicht GemachtВекторный расчет us Made EasyVektor Analysis Leicht GemachtWirtschaftsmathematik Leicht Gemacht

Calculus Made EasyACT Made EasyAccounting Made EasyAerodynamics Made EasyAnalysis Leicht GemachtAnalysis mit PhysikAlgebra Made Easy CX CASAlgebra Made Easy CXAlgebra Leicht Gemacht CX CASAlgebra de Manera FacilAlgebra de Manera Facil CXApplications and Optimizations Made EasyBiology Made EasyBiostatistics AppBusiness Calculus Made EasyBusiness Stats with Physics with de Manera Facil Chemistry Made EasyChemie Leicht GemachtQuimica de Manera FacilCollege Algebra Made Easy CX CASCollege Algebra Made Easy CXComplex Analysis Made EasyConics Made EasyConico de Manera FacilDifferential Equations Made EasyEcuaciones Diferencial de Manera FacilDifferential Gleichungen Leicht GemachtDiscrete Math Made EasyMatematicas Discretas de Manera FacilEconomics Made EasyEinheiten Umwandler mit SchrittenElectrical Engineering Made EasyElectronik Leicht GemachtEngineering Economics Made EasyEngineering Mathematics Made EasyIngenieur Mathematik Leicht GemachtFinance Mad e EasyFinite Math Made EasyGeometry Made EasyGeometrie Leicht GemachtGeometria de Manera FacilLand Surveying Made EasyLinear Algebra Made EasyLinear Algebra de Manera FacilLineare Algebra Leicht GemachtMathematical Economics Made EasyMathematical Economics Made EasyMathematical Economics Made EasyCXMatrix Made EasyNumerical Analysis Made EasyNumber Theory Made EasyProperties Research Made EasyPhysik Made EasyPhysik Leicht Gemacht & Stocks Made EasyPreCalculus Made EasyPreCalculus Made Easy CXPreCalculus Made Easy CXPreCalculo de Manera FacilReal Estate Made EasySAT Made EasySAT Subject Test MathСигналы и системы Made EasyСтатистика и вероятность Made EasyStatistik Leicht GemachtEstadisticas de Manera FacilStatik und Dynamics Made EasyStatik und Dynamik Leicht GemachtStep by Step Equal SolverSolucionador de EcuacionesStep de Manera Facil by Step Unit ConverterThermodynamic Made EasyThermodynamik Leicht GemachtТригонометрия Made EasyTrigonometria de Manera FacilTrigonometrie Leicht GemachtВекторный расчет us Made EasyVektor Analysis Leicht GemachtWirtschaftsmathematik Leicht Gemacht

Введите последние 8 цифр вашего 27-значного идентификатора продукта TI-Nspire.

Находится в разделе 5:Настройки → 4:Статус → О программе

ID может выглядеть так: 1008000007206E210B0 BD92F455 . ПОМОГИТЕ НАЙТИ ID.
Если бы это был ваш ID, вы бы набрали только BD92F455.

или на международном уровне:

В конце оплаты PayPal вам будет отправлено электронное письмо, содержащее ваш ключ и программное обеспечение.


Хотите купить TI-калькулятор?

Получите самые низкие цены на TI-Calculators
(со сравнением цен)

СРАВНИТЕ лучшие цены на Amazon, Ebay, Target, Walmart, Office Max, Best Buy.

Сравните лучшие цены на Amazon, Walmart, Ebay, Target, Best Buy и т. д.
Изучите историю цен калькуляторов за последние несколько месяцев.
Установите оповещение по электронной почте при снижении цен, чтобы получать уведомления.
Сравните различные модели, чтобы найти калькулятор, который лучше всего соответствует вашим потребностям.
Найдите новые, обновленные, восстановленные, подержанные калькуляторы.
Посмотрите обучающие видео и прочитайте руководства по калькуляторам.
Прочтите последние новости о калькуляторах в Интернете.

БЕСПЛАТНАЯ загрузка:
Решатель квадратных уравнений (шаг за шагом)

Загрузите пошаговый решатель квадратных уравнений

. Этот решатель является частью приложения Algebra Made Easy.

— Загрузите бесплатные пробные версии здесь.

— Срок действия пробных и платных приложений неограничен.

— Будущие обновления бесплатны — навсегда!


Онлайн-репетиторство по математике

Получите онлайн-репетиторство.

Репетиторы с отличными оценками по математике будут рады помочь вам.

Получите индивидуальную помощь по математике.

Мы используем Zoom для обучения онлайн, мы шаг за шагом объясняем, как решать математические задачи.

Репетиторы в настоящее время преподают алгебру, алгебру 2, предварительное исчисление, исчисление AP и BC, статистику AP, тригонометрию, дискретную математику.

Репетиторы работали более 10 лет в качестве читателей AP Calculus (те люди, которые оценивают экзамены AP Calculus).

Репетиторы также обучают навыкам сдачи тестов, которые так же важны, как и само содержание.

Наши преподаватели имеют более 20 лет опыта преподавания.

1. урок стоит 50$, после этого 100$ в час.
оптом: 540 долларов за 6 часов, 1000 долларов за 12 часов.

Забронируйте сеанс репетиторства по электронной почте: [email protected]

Для вопросов, заказов и т. д.: ОБЩАЙТЕСЬ С НАМИ.

Первый урок (50 долларов США) Один урок (100 долларов США) Несколько уроков

Коллекция из 88 математических калькуляторов, разделенных по уровню квалификации и типу

Используйте наш бесплатный калькулятор

Мы сотрудничаем с Mathway, чтобы предложить бесплатный онлайн-калькулятор. Обширный список других инструментов исчисления находится ниже.

Содержание

  • Обзор
  • 48 Введение в математические калькуляторы
    • Ограничения
    • Асимптоты
    • Производные
    • Правило продукта
    • Частное правило
    • Скорость изменения
    • Расширенный ряд Тейлора или полином Тейлора
    • Точки перегиба
    • Метод Ньютона
    • Интеграл
  • 40 продвинутых математических калькуляторов
    • Суммы Римана
    • Трапециевидная линейка
    • Разложение на неполные фракции
    • Обратные функции
    • Дифференциальное уравнение
    • Длина дуги
    • Центр масс
    • Последовательности
    • Серия

Обзор

По своей сути, математический факультет Массачусетского технологического института объясняет, что исчисление — это «изучение того, как все меняется». Департамент отмечает, что это важная область исследований, потому что «она дает нам возможность построить относительно простые количественные модели изменений и вывести их последствия».

В Интернете доступно множество ресурсов, которые помогут вам узнать больше об исчислении и его концепциях. Ниже представлена ​​коллекция из 88 калькуляторов, разделенных по уровню квалификации и типу.

48 Введение в вычислительные калькуляторы

Пределы

Изучение пределов будет важной частью вашего изучения исчисления, поскольку они касаются значения, к которому приближается функция, когда вход приближается к определенному значению. Академия Хана предлагает уроки о том, что такое ограничения и как они работают. Ниже приведен набор ресурсов, которые помогут вам лучше понять ограничения:

Ограничение WolframAlpha.com. Результаты включают ваше ограничение, ограничение, нанесенное на график, и расширение ряда.

Лимит Symbolab.com. Четко оформленный и простой в использовании результат включает пошаговое объяснение и возможность увидеть свой лимит на графике.

Предел MathPortal.org — введите свою функцию и проверьте, хотите ли вы найти двусторонний, левосторонний или правосторонний предел. Затем предоставляются четкие результаты.

Ограничение NumberEmpire.com – введите свою собственную функцию или попробуйте один из примеров и получите быстрые и понятные результаты.

Предел SolveMyMath.com — прост в использовании; введите свою функцию, чтобы найти двусторонний, левый или правый предел.

Лимит Calcul.com. Введите свое выражение, и будет предоставлен лимит.

4 Калькуляторы асимптот

MathIsFun.com учит, что асимптота — это «линия, к которой приближается кривая, стремясь к бесконечности». Ниже приведен набор инструментов, которые помогут вам ознакомиться с асимптотами.

Асимптоты WolframAlpha.com. Используйте раскрывающееся меню, чтобы выбрать, какую асимптоту вы хотите найти: наклонную, горизонтальную или вертикальную.

Асимптоты Symbolab.com – введите собственную функцию или выберите один из примеров. Результаты включают краткие пояснения и графическую асимптоту.

Асимптоты EasyCalculation.com. Каждый из этих инструментов включает в себя различные возможные методы, используемые для решения асимптот. Просто введите свое уравнение, и результаты будут включать точку асимптоты, а также асимптоту на графике.

  • Горизонтальная асимптота
  • Вертикальная асимптота

Производные

Как объясняет SOSMath.com, производная часто определяется двумя способами: либо как «наклон кривой», либо как «скорость изменения». Ниже приведен набор ресурсов, которые помогут вам узнать больше о деривативах:

SolveMyMath.com Derivative — попробуйте один из примеров или введите собственное выражение. График производной предоставляется вместе с вашими результатами.

Производные от Calculus-Calculator.com – просты в использовании и предоставляют пошаговые объяснения вместе с вашими результатами.

Производные от WolframAlpha.com. Узнайте больше о деривативах из подробного руководства. Результаты включают графическую производную, ее разложение в ряд, неопределенный интеграл и многое другое.

Derivative-Calculator.net’s Derivative – Простота с пошаговым объяснением, которое дается вместе с вашими результатами.

Производные от Symbolab.com. Четко оформленный и простой в использовании. Вы можете либо ввести собственное выражение, либо использовать один из примеров, чтобы узнать больше о деривативах. Пошаговое объяснение предоставляется вместе с результатами.

MathPortal.org Derivative — следуйте инструкциям, чтобы правильно ввести выражение. Может быть предоставлена ​​первая, вторая или третья производная.

WebMath.com «Найти производную» — предоставлена ​​учебная информация, а результаты включают пошаговое объяснение.

Пошаговые вычисления производных на Calc101.com. Включает пошаговое объяснение того, как найти первую и вторую производные.

Производное от EasyCalculation.com. Следуйте инструкциям, чтобы правильно ввести выражение.

Производная PlanetCalc.com. Чтобы узнать больше о производных, ознакомьтесь с предоставленными правилами дифференцирования и производными общих функций.

Производная Calcul.com — введите свое выражение, и будет предоставлена ​​производная.

Производная от Saltire.com — введите свою функцию, и результаты будут показаны на графике.

Правило произведения

EasyCalculation.com объясняет, что правило произведения — это «метод нахождения производной функции, являющейся произведением двух других функций, для которых существуют производные». Ниже приведены два инструмента, которые используют правило произведения для поиска производной:

Правило продукта WolframAlpha.com – очень простое в использовании, просто введите свою функцию и получите результат.

Правило продукта EasyCalculation.com — введите собственную функцию или используйте один из встроенных примеров. Правило продукта используется для предоставления ваших результатов.

Частное правило

Как объяснили на факультете математики Калифорнийского университета в Дэвисе, частное правило — это «формальное правило для дифференциации задач, в которых одна функция делится на другую».

Частное правило EasyCalculation. com. Некоторая учебная информация предоставляется, чтобы помочь вам лучше понять это правило. Введите свою собственную функцию или попробуйте один из примеров, предоставленных для дополнительной иллюстрации.

Частное правило WolframAlpha.com — введите числитель и знаменатель, чтобы найти производную вашей функции с помощью частного правила.

Скорость изменения

Сайт MathWords.com отмечает, что скорость изменения — это «изменение значения величины, деленное на прошедшее время». Ниже приведены инструменты, которые помогут вам узнать больше о скорости изменения.

Средняя скорость изменения TutorVista.com. Используйте предоставленные пошаговые объяснения, чтобы узнать больше о том, как найти скорость изменения.

Средняя скорость изменений WolframAlpha.com – быстрая, простая в использовании и дает четкие результаты.

Ряд расширения Тейлора или полином Тейлора

Как объясняет MathIsFun.com, ряд Тейлора — это «разложение функции в бесконечную сумму членов». Ниже приведены ресурсы, которые помогут вам узнать больше о рядах Тейлора — понятии, которое часто сбивает с толку студентов, изучающих математику, при первом знакомстве.

Серия Taylor от WolfamAlpha.com. Приведены примеры, показывающие, как использовать этот инструмент для расширения серии на основе определенных критериев. Результаты включают расширение серии, графическое наглядное пособие и многое другое.

Серия Taylor от NumberEmpire.com – содержит краткую учебную информацию. Используйте один из четырех примеров или введите свою собственную функцию. Предоставляются удобные для пользователя результаты и возможность просмотра графического представления.

Расширение серии Taylor от SolveMyMath.com — базовый инструмент, дающий четкие результаты.

Точки перегиба

Как объясняет Wolfram MathWorld, точка перегиба — это «точка на кривой, в которой знак кривизны (т. е. вогнутости) меняется». Ниже приведен инструмент, который поможет вам узнать больше о точках перегиба.

Точки перегиба WolframAlpha.com – просты в использовании, результаты включают точки на графике.

Метод Ньютона

Wolfram MathWorld учит, что метод Ньютона (или Ньютона-Рафсона) — это «алгоритм нахождения корня, в котором используются первые несколько членов ряда Тейлора функции вблизи предполагаемого корня». Ниже приведены инструменты, которые помогут вам научиться использовать метод Ньютона:

Метод Ньютона от Keisan.Casio.com. Предоставляется формула метода Ньютона. Введите свою функцию и ее производную, чтобы получить результаты.

Средство решения уравнений метода Ньютона от Shodor.org – быстрое и простое в использовании, просто введите свою функцию, ее производную, начальное значение «x» и количество знаков после запятой, которые должны быть указаны в вашем ответе, и вы получите результаты. Он также сообщает вам, сколько итераций потребовалось, чтобы получить ваш ответ.

Метод Ньютона-Рафсона от WolframAlpha.com – быстрый и простой; приведена формула.

Метод Ньютона от Maccery.com. Прокрутите вниз до инструмента «Метод Ньютона». Введите свои данные. Результаты будут включать каждую итерацию.

Интегралы

Как объясняет Wolfram MathWorld, интеграл — это «математический объект, который можно интерпретировать как площадь или обобщение площади». Приведенные ниже инструменты помогут улучшить вашу способность работать с интегралами:

Integral от Calculus-Calculator.com — с ним легко работать, он предоставляет пошаговое объяснение вместе с вашими результатами.

Integral от WolframAlpha.com. Узнайте больше об интегралах из предоставленных учебных материалов и примеров. Результаты включают графическое представление, разложение в ряд и неопределенный интеграл.

Интеграл Integral-Calculator.com — содержит примеры, которые помогут вам начать работу.

Integral от Symbolab.com — четко оформленный и включает пошаговое объяснение с результатами.

Integral MathPortal.org. Следуйте предоставленным инструкциям, чтобы убедиться, что вы вводите свои данные правильно. Используйте кнопку «Создать пример», чтобы узнать больше о том, как работают интегралы.

Integral от NumberEmpire.com – используйте один из четырех предоставленных примеров или введите свою собственную функцию. Результаты легко интерпретировать.

SolveMyMath.com Integral – прост в использовании и дает четкие результаты.

WebMath.com Решите неопределенный интеграл — отлично подходит для тех, кто только начинает работать с интегралами. Лучше всего использовать этот инструмент только с базовыми интегралами.

Экспоненциальный интеграл Keisan.Casio.com. Введите значение «x», чтобы начать. Результаты включают вашу функцию на графике и двухэтапное объяснение.

Integral от CalCul.com. Введите свое выражение, и вам будут предоставлены результаты.

Логарифмический интеграл Had2Know.com. Учебное пособие предназначено для укрепления ваших знаний об интегралах. Введите значение «x» и получите четкие результаты.

Экспоненциальный интеграл MiniWebTool. com. Определяющая формула предоставляется в качестве справочной информации. Введите значение «x», чтобы получить результаты.

40 Усовершенствованные вычислительные калькуляторы

Суммы Римана

Как объясняет MathOpenRef.com, сумма Римана — это «метод аппроксимации общей площади под кривой на графике, также известный как интеграл». Ниже приведен набор ресурсов, которые помогут вам лучше понять суммы Римана.

Сумма Римана MathWorld.Wolfram.com — введите свои данные, чтобы увидеть сумму Римана на графике. Поэкспериментируйте с введенными данными, чтобы увидеть, как изменится график.

Сумма Римана на сайте EMathHelp.net — проста в использовании и включает пошаговое объяснение ваших результатов.

Апплет Riemann Sums от IntMath.com – Предоставляется учебная информация. Выберите функцию в раскрывающемся меню, чтобы увидеть, как она отображается на графике. Отрегулируйте ползунки, чтобы увидеть, как сумма Римана изменяется на графике.

Правило трапеций

MathWords. com объясняет, что правило трапеций « метод приближения определенного интеграла с использованием линейных приближений f ». Приведенные ниже инструменты помогут вам научиться пользоваться правилом трапеций.

Трапециевидное правило NastyAccident.com. Следуйте инструкциям, чтобы ввести свои данные. Результаты включают пошаговое объяснение.

Трапециевидное правило EMathHelp.net — обеспечивает пошаговое объяснение ваших результатов.

Трапециевидное правило EasyCalculation.com. Узнайте больше о трапециевидном правиле из предоставленной учебной информации. Следуйте инструкциям, чтобы убедиться, что вы вводите свои данные правильно.

Разложение частичной фракции

Как поясняет сайт PurpleMath.com, разложение на неполные дроби — это «процесс, который начинается с упрощенного ответа и разбирается на части, или «разложение» окончательного выражения на исходные полиномиальные дроби». Ниже приведен набор ресурсов, которые помогут вам лучше понять разложение на неполные дроби.

Разложение на неполные дроби от WolframAlpha.com – Просто и понятно, просто введите числитель и знаменатель, чтобы получить результат.

Пошаговое определение частичных дробей на Calc101.com. Введите свое выражение (или используйте предоставленный пример), после чего будет предоставлено пошаговое объяснение для нахождения частичной дроби.

Частичные дроби на сайте QuickMath.com — быстро и легко использовать, просто введите функцию, чтобы найти неполную дробь. Предусмотрены базовая и расширенная версии.

Частичные дроби Symbolab.com — введите свое выражение или используйте один из предоставленных примеров. Пошаговое объяснение будет предоставлено с результатами.

Обратные функции

Как объясняет Wikipedia.org, обратная функция «это функция, которая «обращает» другую функцию». Ниже приведен набор инструментов, которые помогут вам лучше понять обратные функции.

Обратная функция Symbolab.com — четко спроектированная, простая в использовании и предоставляет пошаговое объяснение с результатами. Нажмите «График», чтобы увидеть обратную функцию на графике.

Обратная функция WolframAlpha.com. Достаточно простая, чтобы проиллюстрировать основы, результаты включают вашу обратную функцию в виде графика.

Обратная функция NumberEmpire.com — выберите один из четырех примеров или введите свою собственную функцию, чтобы получить обратную функцию.

Обратная функция AnalyzeMath.com — нажмите кнопку «Показать», и этот ресурс проведет вас через четырехэтапный процесс поиска обратной функции.

Обратная функция CalculatorSoup.com. Используйте раскрывающееся меню, чтобы выбрать, какую функцию вы хотите найти. Затем введите значение «x», чтобы получить результаты.

Обратная функция Keisan.Casio.com — введите значение «x», и будут предоставлены обратные гиперболические функции.

Обратная функция Gyplan.com. Используйте раскрывающееся меню, чтобы выбрать тип обратной функции, которую вы хотите найти, а затем введите значение «x», чтобы получить результаты.

Дифференциальное уравнение

Как объясняет Wolfram MathWorld, дифференциальное уравнение — это «уравнение, в которое входят производные функции, а также сама функция». Ниже приведены несколько инструментов, которые помогут вам узнать больше о дифференциальных уравнениях:

Дифференциальные уравнения WolframAlpha.com – Используйте для решения различных видов дифференциальных уравнений. Результаты включают в себя решение, графики образцов отдельных решений, графическое изображение семейства образцов решений и многое другое.

Обыкновенные дифференциальные уравнения от Symbolab.com. Четко разработанные и простые в использовании результаты включают пошаговые объяснения. Введите собственное уравнение или проведите эксперимент, используя предоставленные примеры.

Решатель дифференциальных уравнений первого порядка от Math-CS.Gordon.edu — от факультета математики и компьютерных наук Гордонского колледжа, решатель уравнений поставляется с некоторой учебной информацией. Результаты включают график решения.

Однородные дифференциальные уравнения EasyCalculation.com — быстро, легко в использовании и дает четкие результаты.

Метод Эйлера от MathScoop.com — использует метод Эйлера для решения вашего уравнения. Результаты включают таблицу Эйлера и график точек Эйлера.

Метод Эйлера от Keisan.Casio.com. Включена необходимая формула, и с вашими результатами создается таблица Эйлера.

Средство решения дифференциальных уравнений второго порядка от Had2Know.com. Узнайте больше о решении дифференциальных уравнений из предоставленной учебной информации и объясненных случаев.

Длина дуги

MathWords.com учит, что длина дуги — это длина кривой или линии. Ниже приведен набор ресурсов, которые помогут вам определить длину дуги.

Длина дуги 1728.org — выберите, что вы хотите решить, затем введите известные значения. Ваш результат будет предоставлен.

Полная дуга окружности на сайте HandyMath.com. Введите два известных значения, чтобы найти радиус, длину, ширину, высоту, апофему, угол и площадь сегмента дуги или окружности.

Дуга окружности AJDesigner.com – даны круговая диаграмма с метками и формула длины дуги. Введите радиус и центральный угол, чтобы получить результат.

Длина дуги WolframAlpha.com. Этот ресурс будет выполнять несколько функций, связанных с определением длины дуги, и предоставляет примеры для каждой из них, которые помогут вам начать работу.

Длина дуги на сайте TutorVista.com. Предоставляется пошаговое объяснение того, как найти длину дуги, а также примеры с пояснениями и результатами.

Интерактивная длина дуги MathOpenRef.com — перетащите точку A или точку B, чтобы увидеть, как изменяется длина дуги.

Длина дуги на сайте Flexibility.com — выберите параметр для определения длины дуги на основе известных значений. Размеченная круговая диаграмма дается в качестве наглядного пособия.

Длина дуги PlanetCalc.com. Предоставляется маркированная круговая диаграмма и формулы. Введите радиус и угол, чтобы найти длину дуги и другие свойства, такие как площадь, длина хорды и периметр.

Длина дуги EasyCalculation.com. Введите радиус и угол, и будет предоставлена ​​длина дуги.

Центр масс

MathWords.com предоставляет формулы для нахождения центра масс. Ниже приведен набор инструментов, которые помогут укрепить ваше понимание центра масс.

Центр масс TutorVista.com — введите «различное значение масс» и «расстояние соответствующих масс», чтобы найти центр масс.

Центр масс Calculator.Swiftutors.com – Предоставляет учебную информацию, очень простую и удобную для любого студента.

LearningAboutElectronics.com’s Center of Mass – Предоставляются обучающая информация, маркированная схема и инструкции по использованию инструмента. Введите все известные массы и их соответствующие расстояния, чтобы найти центр масс.

Последовательности

Как объясняется в онлайн-заметках Пола по математике, последовательность — это «список чисел, записанных в определенном порядке». Приведенный ниже инструмент поможет вам узнать больше о последовательностях:

Последовательности WolfamAlpha. com. Приведены примеры, показывающие, как использовать этот ресурс на основе различных критериев последовательностей. Результаты включают графическое представление, таблицу значений и рядовые представления.

Серия

Как объясняет MathOpenReference.com, серия — это «сумма некоторого набора членов последовательности». Используйте приведенные ниже ресурсы, чтобы укрепить свое понимание сериалов.

Серия NumberEmpire.com – используйте один из четырех предоставленных примеров или введите собственное выражение. Результаты легко интерпретировать, и вы можете отредактировать свое выражение.

Геометрическая серия MathScoop.com – просто и быстро, просто введите свои значения, и сумма будет предоставлена.

Геометрическая серия CalCul.com – отрегулируйте значения переменных с помощью стрелок, и результат будет предоставлен мгновенно.

Калькулятор производных

Калькулятор производных с шагами

Калькулятор производных (также известный как калькулятор дифференцирования) используется для определения скорости изменения заданной функции по отношению к ее независимой переменной. Функция может быть постоянной, линейной, полиномиальной, квадратичной полиномиальной и т. д.

Дифференциальный калькулятор распознает функцию и вычислит ее производную. Существует три вида дифференциала.

  • Явное дифференцирование
  • Неявное дифференцирование
  • Частичное дифференцирование

Этот решатель производных оценивает явное дифференцирование любой функции одним щелчком мыши.

Как работает этот калькулятор дифференциации?

Для решения задач явного дифференцирования выполните следующие действия.

  • Введите постоянную, линейную или полиномиальную функцию в поле ввода.
  • Выберите независимую переменную, такую ​​как x, y, z, u, v, t или w.
  • Используйте значок клавиатуры для ввода арифметических и других символов, таких как √, +, -, e и т. д.
  • Введите количество производных, например 1 для первой производной или 2 для второй производной.
  • Нажмите кнопку вычислить под полем ввода, чтобы получить результаты.
  • Нажмите кнопку сброса , чтобы войти в другую функцию.
  • Вы можете нажать показать больше для просмотра пошагового решения.

Что такое производная?

Производная — это скорость изменения функции по отношению к ее переменной. Производные лежат в основе решения задач дифференциальных уравнений и исчисления.

Процесс, обратный дифференцированию, известен как интеграл.

Правила дифференцирования

Вот некоторые основные правила дифференцирования, упомянутые ниже.

9{n-1}\)
Правило сумм \(\frac{d}{dx}\left(u\left(x\right)+v\left(x\right)\right)=\ frac{d}{dx}\left(u\left(x\right)\right)+\frac{d}{dx}\left(v\left(x\right)\right)\)
Правило разности \(\frac{d}{dx}\left(u\left(x\right)-v\left(x\right)\right)=\frac{d}{dx}\left(u \left(x\right)\right)-\frac{d}{dx}\left(v\left(x\right)\right)\)
Правило произведения \(\frac{d} {dx}\left(u\left(x\right)\cdot v\left(x\right)\right)=v\left(x\right)\frac{d}{dx}\left(u\left (x\право)\право)+u\лево(x\право)\frac{d}{dx}\left(v\left(x\right)\right)\) 92

Ссылки

  • Что такое производная? | Британская энциклопедия.
  • Правила дифференцирования. Исследование.com | Пройдите онлайн-курсы. Заработайте кредит колледжа.

Калькулятор производных — найти производную функции бесплатно в Интернете

Введение в калькулятор производных

Калькулятор производных — это онлайн-инструмент, который помогает в расчетах основных понятий вывода и дифференцирования. С этим калькулятором можно работать онлайн с помощью простых кликов. Калькулятор производной функции прост в использовании и содержит простые инструкции, которые легко понять.

Этот калькулятор дает полные результаты вывода после процесса расчета. Калькулятор производной функции поможет вам найти результаты производной шаг за шагом и упростит изучение производной. Лучшее свойство решателя производных состоит в том, что он бесплатный и помогает быстрее находить решения.

Важность и преимущества использования онлайн-калькулятора дифференциации

В мире, полном технологий, ручные расчеты и решения считаются сложным процессом. Поэтому Дифференциальный калькулятор сделал решение для решения производных с помощью этого калькулятора. Производный решатель имеет некоторые из основных преимуществ и важности его использования, а именно:

  1. Калькулятор формулы производной помогает более точно вычислить производную.
  2. Это помогает тратить ваше драгоценное время на вычисление производной вручную.
  3. Этот калькулятор поможет вам выполнить каждый шаг, который можно решить с помощью этого калькулятора.
  4. Этот калькулятор обеспечит выбор мощности функции и возможность выбора переменных по вашему выбору.
  5. Этот калькулятор является бесплатным и дает вам все шаги результата шаг за шагом.
  6. Использование этого калькулятора сократит время вычислений, а также позволит более точно рассчитать результат.
  7. Этот калькулятор быстро рассчитает производную функцию.

Как найти основной калькулятор дифференциации?

В Интернете есть много калькуляторов производных функций, но чтобы найти основной калькулятор дифференцирования с шагами, обладающими вышеуказанными свойствами, просто выполните следующие шаги:

  1. Прежде всего введите в Google основное ключевое слово этого калькулятора — «Производный калькулятор». Google поможет вам в поиске этого калькулятора и приведет вас к основной производной функции калькулятора.
  2. Google также покажет вам различные другие результаты, связанные с расчетом дифференциации, но все зависит от вас, что вы там выберете.
  3. Другой способ найти этот инструмент — найти родительский веб-сайт, на котором есть этот калькулятор. Вы можете напрямую ввести URL-адрес в браузере или выполнить поиск на веб-сайте CALCULATORES. На веб-сайте есть множество инструментов интеграции, которые могут помочь вам учиться, понимать и практиковать онлайн.

Результаты какого типа формируются калькулятором производных?

Калькулятор функции производной с шагами более точно решает функции и дает правильный ответ производной. Он решает уравнения шаг за шагом и предоставляет простые для понимания инструкции. Результаты, полученные этим калькулятором, могут быть подсчитаны любым другим источником, и все результаты будут точными или достоверными.

Уместно ли использование калькулятора производных?

Калькулятор производной функции — важный инструмент, упрощающий расчет производной. Это также помогает экономить время и энергию. Использование этого калькулятора считается более подходящим из-за его пошаговых ответов и простых для понимания инструкций. Этот калькулятор имеет простые шаги и простые инструкции для понимания. У него есть основные требования, которым можно следовать, а затем он дает наилучшие решения более подходящим образом.

Различные этапы использования калькулятора производных

Калькулятор производной с шагами упростил использование этого калькулятора. Различные шаги, которые помогают ввести функцию в калькулятор и помогают найти результат, можно объяснить следующим образом:

  1. Первым шагом этого решателя производных является ввод функции в калькуляторе.
  2. Загрузите пример из калькулятора или у него также есть клавиатура с правой стороны калькулятора. Таким образом, вы также можете ввести функцию в соответствии с вашими потребностями.
  3. Выберите переменные в калькуляторе, поскольку он имеет несколько переменных, и убедитесь, что выбранные переменные отображаются в вашей функции.
  4. Выберите время функций 1, 2, 3 …. в Интернете.
  5. Последним шагом является нажатие кнопки «Рассчитать», чтобы быстро найти производную функцию.

Каковы причины выбора калькулятора производных?

Есть несколько причин для выбора этого калькулятора формулы производной с шагами. Этот калькулятор имеет лучшие и самые простые в использовании функции. Это дает наилучшие результаты деривации в течение короткого времени. Он обеспечивает решение каждой проблемы быстрее. Он имеет пошаговые инструкции и точные результаты, которые легко понять. Этот бесплатный калькулятор поможет вам найти производную функцию с помощью простых шагов и поможет легче изучить производную функцию.

Алан Уокер

Последнее обновление 02 июня 2022 г.

Изучает математические науки и технологии. Технический гик и автор контента. Помешанный на Википедии, который хочет знать все. Любит путешествия, природу, чтение. Математика и технологии сделали свое дело, и теперь пришло время извлечь выгоду из этого.

Вычисление производной в Excel

Несколько недель назад я писал о вычислении интеграла данных в Excel. На этой неделе я хочу изменить направление и показать, как вычислить производную в Excel. Как и при численном интегрировании, в Excel есть два способа выполнить этот расчет:

  • Производные табличных данных на листе
  • Производная функция с использованием VBA (или Visual Basic для приложений)

В этом посте я сосредоточусь на вычислении производных табличных данных, а пост о вычислении того же с помощью VBA появится позже.

Содержание

  • Как дифференцировать в Excel
  • Выработка Excel Formula с использованием метода конечных разностей
  • Пример: Рассчитайте производную в Excel
    • Как рассчитать скорость в Excel
    • .
    • Результаты
    • Подведение итогов

    Дифференциация в Excel

    Это вид расчета производной, который обычно выполняется на основе экспериментальных данных. Это может быть особенно полезно, когда вы не можете напрямую измерить интересующую величину, но можете измерить ее подынтегральную функцию.

    Классический пример, конечно, это положение и скорость:

    Скажем, например, вы провели какой-то эксперимент, в котором было трудно получить скорость напрямую. Вместо этого вы измерили положение в разное время, t . Вы можете импортировать данные в Excel и рассчитать скорость как производную от положения по времени.

    Формула производной Excel с использованием метода конечных разностей

    Метод, используемый для выполнения этого вычисления в Excel, представляет собой метод конечных разностей.

    Чтобы использовать метод конечных разностей в Excel, мы вычисляем изменение «y» между двумя точками данных и делим на изменение «x» между теми же точками данных:

    Это называется односторонней оценкой. , потому что он учитывает наклон данных только с одной стороны интересующей точки. Приведенная выше формула возвращает тот же результат, что и функция НАКЛОН в Excel, поэтому мы могли бы ее использовать. Однако это не предпочтительный метод.

    Лучшей оценкой будет вычисление среднего уклона в интересующей точке путем усреднения уклона непосредственно до и после этой точки.

    Итак, если бы мы хотели найти наклон в точке y 2 (z), мы могли бы использовать это вычисление:

    Эта формула производной известна как центральная конечная разность.

    Пример: вычисление производной в Excel

    Давайте рассмотрим, как вычислить производную в Excel на примере. Мы можем использовать данные о положении, которые были рассчитаны путем интегрирования данных о скорости в предыдущем посте, и использовать их для расчета как скорости, так и ускорения. В качестве проверки сравним рассчитанные данные ускорения с исходными данными ускорения.

    Чтобы упростить задачу, я скрыл старые данные об ускорении и скорости. Мы посмотрим, как они сравниваются в конце.

    Как рассчитать скорость в Excel

    Сначала я вычисляю скорость как производную данных о положении, используя вышеприведенное уравнение конечных разностей. Поскольку нам нужны y3 и y1, я начинаю расчет в ячейке E5 и заполняю ее.

    Как вычислить первую производную в Excel

    Затем, используя вычисленную скорость, я могу рассчитать ускорение (которое является первой производной скорости) тем же методом. На этот раз расчет начинается в строке 6.

    Результаты

    Теоретически, если мы продифференцируем данные, полученные интегрированием, мы должны вернуться к исходным данным. Конечно, все численные методы вносят какую-то ошибку в данные.

    Но насколько серьезна ошибка? Давайте сравним.

    В этом случае мы можем увидеть некоторые небольшие различия между исходными данными ускорения и данными, полученными дифференцированием в Excel. Есть также некоторые небольшие различия в двух наборах данных скорости. К счастью, ошибка при численном дифференцировании не накапливается, в отличие от численного интегрирования.

    Таблицы данных не являются идеальным способом изучения этих данных, поэтому давайте посмотрим на графики:

    Это трудно увидеть, потому что две линии лежат друг над другом, но для всех практических целей скорости идентичны .

    Как насчет ускорения?

    Здесь мы видим, что в периоды постоянного или постоянного ускорения два набора данных очень похожи. Однако при наличии разрыва в данных об ускорении (т. е. в моменты времени 0,1, 0,45, 0,5, 0,7 и 0,75 с) ускорение, полученное дифференцированием (оранжевый цвет), не соответствует исходным данным ускорения (синий цвет).

    Это связано с уравнением, которое мы использовали для дифференцирования. Помните, как мы получили производную в точке путем 90 361 усреднения 90 362 наклона по обе стороны от этой точки? Мы видим результаты этого здесь.

    Подведение итогов

    Если вы следовали инструкциям, поздравляем! Вы только что выполнили численное дифференцирование с помощью Excel. Вычислить производную в Excel не так уж сложно, если вы знаете, как это сделать.

     

    исчисление — Как проверить правильность уравнения производной?

    спросил

    Изменено 11 лет, 6 месяцев назад

    Просмотрено 26 тысяч раз

    $\begingroup$

    Я могу вычислить производную функции, используя правило произведения, правило цепочки или правило частного.

    Однако, когда я нахожу результирующую производную функцию, у меня нет возможности проверить правильность моего ответа!

    Как проверить правильность вычисленного уравнения производной? (т.е. я не ошибся с факторингом или с одним из правил).

    У меня есть графический калькулятор.

    Спасибо!

    • исчисление

    $\endgroup$

    4

    $\begingroup$

    Вы можете подтвердить это численно, аппроксимировав его значение в определенных точках. По определению: $$f\,'(x)=\lim_{\Delta x \to 0}{\frac{f(x + \Delta x) — f(x)}{\Delta x}}$$

    Пример: $$\sin(x)’=\cos(x)$$ Пусть $\Delta x=0,000001$ и $x=1$ $$\sin(1)’\приблизительно\frac{\sin(1+0,000001)-\sin(1)}{0,000001}\приблизительно\frac{0,841471525 — 0,841470985}{0,000001}\приблизительно0,54030189$$

    Теперь ваша производная: $$\cos(1)\ приблизительно 0,54030231$$

    Что очень близко. Чем меньше $\Delta x$, тем ближе результат. Выберите несколько точек на оси X и проверьте их таким образом.

    Вероятно, вы можете определить пользовательскую функцию с помощью калькулятора и построить ее график: $$ \mathrm{error}(x)=\left|\frac{f(x+0,0000001)-f(x-0,0000001)}{0,0000002 }-f\,'(x)\right|$$

    Учтите, что ошибку аппроксимации необходимо сравнить со значением вашей производной в этой точке.

    $\endgroup$

    1

    $\begingroup$

    Для любой конкретной производной вы можете попросить компьютер проверить ваш результат, как предлагают несколько других ответов.

    Однако, если вы хотите быть самодостаточным при использовании деривативов (для экзамена или другой работы), я рекомендую много целенаправленной практики. Большинство учебников по математическому анализу включают ответы на задачи с нечетными номерами в конце книги, и если вы введете запрос «рабочий лист с производными», вы найдете множество списков задач в Интернете.

    Проработайте список из не менее 20 задач и проверьте свои ответы. Если вы правильно ответили менее чем на 80% или 90%, значит, вам нужно больше практики. Вот самая важная часть: Отследите свои ошибки. Остерегайтесь их в будущем и убедитесь, что вы понимаете правильный путь. Обратите также внимание на упрощение ответов, потому что многие люди делают алгебраические ошибки после правильного исчисления.

    Правила, которые у вас есть, — лучший способ использовать эти производные, вам просто нужно уметь их правильно использовать.

    $\endgroup$

    $\begingroup$

    Многие производные задачи можно решить несколькими способами. Один из способов проверить свою работу — попробовать оба способа и посмотреть, получится ли у вас одно и то же. 93}$

    Возможно, это слишком простой пример, но я надеюсь, что вы уловили идею.

    $\endgroup$

    $\begingroup$

    У меня есть TI-84, и у него есть функция числовой производной. Его можно найти, нажав кнопку MATH, и это 8-я функция в списке.

    nDeriv( появится на экране. Затем вы можете ввести исходную функцию, запятую, переменную в исходной функции (вероятно, $x$, если вы не сохранили что-то еще), запятую и значение x, которое вы хотите для калькулятор для подключения к производной.93-5Х+9,Х,10)

    Затем вы подставляете то же значение x к предполагаемой производной, которую вы проверяете, и решаете ее вручную.

    Если значения совпадают, ваша производная верна!

    $\endgroup$

    $\begingroup$

    Вы можете проверить определенные значения, такие как седловые точки, экстремальные точки и локальные минимумы/максимумы, установив первую производную равной нулю/проведя дальнейшее вычисление и проверив эти производные.

Добавить комментарий

Ваш адрес email не будет опубликован. Обязательные поля помечены *